ChaseDream

标题: 【每日逻辑练习第二季【2-3】 [打印本页]

作者: bananazoo    时间: 2011-12-12 00:31
标题: 【每日逻辑练习第二季【2-3】
今天发晚了~电脑坏了,囧~开不了机。。。希望里面的一切都在~用同学的电脑来发作业~~~
不过我们不会被任何困难打到,对不对~大家加油!
【精练】

4. Amphibian populations are declining in numbers
worldwide. Not coincidentally, the earth’s ozone
layer has been continuously depleted throughout the
last 50 years. Atmospheric ozone blocks UV-B, a
type of ultraviolet radiation that is continuously
produced by the sun, and which can damage genes.
Because amphibians lack hair, hide, or feathers to
shield them, they are particularly vulnerable to UV-B
radiation. In addition, their gelatinous eggs lack the
protection of leathery or hard shells. Thus, the
primary cause of the declining amphibian population
is the depletion of the ozone layer.
Each of the following, if true, would strengthen the
argument EXCEPT:
(A) Of the various types of radiation blocked by
atmospheric ozone, UV-B is the only type
that can damage genes.
(B) Amphibian populations are declining far more
rapidly than are the populations of
nonamphibian species whose tissues and
eggs have more natural protection from
UV-B.
(C) Atmospheric ozone has been significantly
depleted above all the areas of the world in
which amphibian populations are declining.
(D) The natural habitat of amphibians has not
become smaller over the past century.
(E) Amphibian populations have declined
continuously for the last 50 years.

【逻辑链】
17.
At present the Hollywood Restaurant has only standard-height tables.  However, many customers come to watch the celebrities who frequent the Hollywood, and they would prefer tall tables with stools because such seating would afford a better view of the celebrities.  Moreover, diners seated on stools typically do not stay as long as diners seated at standard-height tables.  Therefore, if the Hollywood replaced some of its seating with high tables and stools, its profits would increase.

The argument is vulnerable to criticism on the grounds that it gives reason to believe that it is likely that
(A) some celebrities come to the Hollywood to be seen, and so might choose to sit at the tall tables if they were available
(B) the price of meals ordered by celebrities dining at the Hollywood compensates for the longer time, if any, they spend lingering over their meals
(C) a customer of the Hollywood who would choose to sit at a tall table would be an exception to the generalization about lingering
(D) a restaurant's customers who spend less time at their meals typically order less expensive meals than those who remain at their meals longer
(E) with enough tall tables to accommodate all the Hollywood's customers interested in such seating, there would be no view except of other tall tables


18.
The Calex Telecommunications Company is planning to introduce cellular telephone service into isolated coastal areas of Caladia, a move which will require considerable investment.  However, the only significant economic activity in these areas is small-scale coffee farming, and none of the coffee farmers make enough money to afford the monthly service fees that Calex would have to charge to make a profit.  Nevertheless, Calex contends that making the service available to these farmers will be profitable.

Which of the following, if true, provides the strongest support for Calex's contention?
(A) Currently, Caladian coffee farmers are forced to sell their coffee to local buyers at whatever price those buyers choose to pay because the farmers are unable to remain in contact with outside buyers who generally offer higher prices.
(B) In the coastal areas of Caladia where Calex proposes to introduce cellular telephone service, there is currently no fixed-line telephone service because fixed-line companies do not believe that they could recoup their investment.
(C) A cellular telephone company can break even with a considerably smaller number of subscribers than a fixed-line company can, even in areas such as the Caladian coast, where there is no difficult terrain to drive up the costs of installing fixed lines.
(D) Calex bases its monthly fees for cellular telephone service in a given region partly on the cost of installing the necessary equipment to provide the service there.
(E) Calex has for years made a profit on cellular telephone service in Caladia's capital city, which is not far from the coastal region.


19.
With a record number of new companies starting up in Derderia and with previously established companies adding many jobs, a record number of new jobs were created last year in the Derderian economy.  This year, previously established companies will not be adding as many new jobs overall as such companies added last year.  Therefore, unless a record number of companies start up this year, Derderia will not break its record for new jobs created.

Which of the following is an assumption on which the argument relies?
(A) Each year, new companies starting up create more new jobs overall than do previously established companies.
(B) Companies established last year will not add a greater number of jobs overall this year than they did last year.
(C) This year, the new companies starting up will not provide substantially more jobs per company than did new companies last year.
(D) This year, the overall number of jobs created by previously established companies will be less than the overall number of jobs lost at those companies.
(E) The number of jobs created in the Derderian economy last year was substantially larger than the number of jobs lost last year.


20.
During the past year, Pro-Tect Insurance Company's total payout on car-theft claims has been larger than the company can afford to sustain.  Pro-Tect cannot reduce the number of car-theft policies it carries, so it cannot protect itself against continued large payouts that way.  Therefore, Pro-Tect has decided to offer a discount to holders of car-theft policies whose cars have antitheft devices.  Many policyholders will respond to the discount by installing such devices, since the amount of the discount will within two years typically more than cover the cost of installation.  Thus, because cars with antitheft devices are rarely stolen, Pro-Tect's plan is likely to reduce its annual payouts.

In the argument above, the two portions in boldface play which of the following roles?
(A) The first and the second are both evidence offered by the argument as support for its main conclusion.
(B) The first presents a problem a response to which the argument assesses; the second is the judgment reached by that assessment.
(C) The first is the position the argument seeks to establish; the second is a judgment the argument uses to support that position.
(D) The first is a development that the argument seeks to explain; the second is a prediction the argument makes in support of the explanation it offers.
(E) The first presents a development whose likely outcome is at issue in the argument; the second is a judgment the argument uses in support of its conclusion about that outcome.

(A) The first and the second are both evidence offered by the argument as support for its main conclusion.
(B) The first presents a problem a response to which the argument assesses; the second is the judgment reached by that assessment.
(C) The first is the position the argument seeks to establish; the second is a judgment the argument uses to support that position.
(D) The first is a development that the argument seeks to explain; the second is a prediction the argument makes in support of the explanation it offers.
(E) The first presents a development whose likely outcome is at issue in the argument; the second is a judgment the argument uses in support of its conclusion about that outcome.

作者: bananazoo    时间: 2011-12-12 00:32
逻辑链答案:C A C B选中前面的看到答案~
精练解析:
This question is much more difficult than the previous question, in part because
one of the wrong answer choices is very attractive.
The conclusion of the argument is a causal statement that the depletion of the
ozone layer is the primary cause of the declining amphibian population:
DO = depletion of the ozone layer
DA = decline of amphibian population
C                E
DO -----> DA
This conclusion is based on the fact that the ozone layer blocks harmful UV-B
radiation, which amphibians are vulnerable to in both adult and egg form.
Although the argument mentions UV-B radiation, which may sound impressive,
the structure of the reasoning is easy to follow and no knowledge of the
radiation is needed. The conclusion is clearly stated and easy to spot due to the
indicator “thus.” The question stem is a StrengthenX and therefore the four
incorrect answers will each strengthen the argument. As with the previous
question, look for answers that fit the five causal strengthening answer types
discussed earlier.
Answer choice (A): This is the correct answer. The answer fails to shed any
light—positive or negative—on the connection between the ozone depletion
and the amphibian population decline. Because the argument is concerned with
the damage done by UV-B radiation, the fact that UV-B is the only damaging
type of radiation blocked by ozone is irrelevant.
Answer choice (B): This answer choice strengthens the argument by showing
that when the cause is absent in nonamphibian populations, the effect does not
occur (Type C).
Answer choice (C): This answer strengthens the argument by showing that the
areas of ozone depletion and amphibian decline match each other, thereby
affirming the data used to make the conclusion (Type E).
Answer choice (D): This was the answer most frequently chosen by test takers.
This answer choice strengthens the argument by eliminating an alternate cause
for the effect (Type A). Had the natural habitat become smaller over the years
(from say, human encroachment or climatic change) then that shrinkage would
have offered an alternate explanation for the decline in the amphibian
population. By eliminating the possibility of habitat shrinkage, the stated cause
in the argument is strengthened.
Answer choice (E): This answer strengthens the argument by showing that the
decline of the amphibians has mirrored the decline of the ozone layer, thereby
affirming the data used to make the conclusion (Type E).

作者: bananazoo    时间: 2011-12-12 00:41
自己占楼~
大冬天的坐暖暖的沙发!
好久没做作业了~惭愧下,最近事情乱七八糟的,诶~,慢慢理顺吧~
从明天做小分队作业开始~
加油!

17
题目到底是不是削弱?还是加强呢?


Theargument is vulnerable to criticism on the grounds that it gives reason tobelieve that it is likely that削弱)

BackgroundH R has onlystandard-height tables.

P: in order to have a better view of the celebrities,customers prefer tall tables.

  Moreover,the time a dinner stay on such table will be less.

恩,这个酒店的位置看不到社会名流?

还有就是moreover里的提示,在这样的桌上的dinner不会长久,会不会也不贵?


(A) some celebrities come to the Hollywood to beseen, and so might choose to sit at the tall tables if they were available这个选项的意思?

(B) the price of meals ordered by celebritiesdining at the Hollywood compensates for the longer time, if any, they spendlingering over their meals(???看不懂)

(C) a customer of the Hollywood who would choose to sit at a tall tablewould be an exception to the generalization about lingering(加强)

(D)a restaurant's customers who spend less time at their meals typically orderless expensive meals than those who remain at their meals longer ----- right

(E) with enough tall tables to accommodate all theHollywood's customers interested in such seating, there would be no view exceptof other tall tables(也是削弱。。。诡辩?)


18

(A)Currently, Caladian coffee farmers are forced to sell their coffee to localbuyers at whatever price those buyers choose to pay because the farmers areunable to remain in contact with outside buyers who generally offer higherprices.


就是说,如果他们装了电话这类的东西~就会有机会和外界联系,收入增加,然后就有钱支付给这个公司,然后C公司就会盈利了~


(B) In the coastal areas of Caladia where Calexproposes to introduce cellular telephone service, there is currently nofixed-line telephone service because fixed-line companies do not believe thatthey could recoup their investment.(和你盈不盈利无关)

(C) A cellular telephone company can break evenwith a considerably smaller number of subscribers than a fixed-line companycan, even in areas such as the Caladian coast, where there is no difficultterrain to drive up the costs of installing fixed lines.???

(D)Calex bases its monthly fees for cellular telephone service in a given regionpartly on the cost of installing the necessary equipment to provide the servicethere.

(E) Calex has for years made a profit on cellulartelephone service in Caladia's capital city, which is not far from the coastalregion.无关

点评:这题觉得自己A,D选不出来,d的意思是不是说fee可以从与这个service有关的领域来?还是从别处来?A的话,觉得蛮对的,就是有些绕远了来赚钱的感觉。。。。

作者: fox0923    时间: 2011-12-12 01:10
在bana楼下~~占位子~~bana明天早上再回去看看电脑里的东西,可能就是死机了而已,不用担心,别忘了早睡觉~-------------------------------------------------------------------
精练------------------55s-----------------strengthen except
B: The A. population has been declined.
P1: The ozone blocks UV-B, which produced by sun and damage the genes.
P2: Since the A. people lack of hair and other features.
C: So the reason of decline of population must be the depletion of ozone layer.
Prephrase: B. population's decline is due to the depletion of ozone layer.


Analysis:
(A) Of the various types of radiation blocked by
atmospheric ozone, UV-B is the only type
that can damage genes.---------------------------------------------this choice is wrong because it sounds to restate the premise and there's no correlation between ozone layer and the decline of population. 但和D相比较还是觉得A有点support的感觉,所以选了D。

(B) Amphibian populations are declining far more
rapidly than are the populations of
nonamphibian species whose tissues and
eggs have more natural protection from
UV-B.-------------------------------------------------------------------support that the non-A species are protected by ozone layer, yet the A specie is not.
(C) Atmospheric ozone has been significantly
depleted above all the areas of the world in
which amphibian populations are declining.---------------------IF ozone layer is depleted above areas where A. specie live, then it's very possible that UV-B killed them. support.
(D) The natural habitat of amphibians has not
become smaller over the past century.---------------------------the natural habitat of A specie has not changed or become larger is irrelevant.
(E) Amphibian populations have declined
continuously for the last 50 years.---------------------------------support.



做错了,应该选A,我选D了。


错误分析:
DO = depletion of the ozone layer
DA = decline of amphibian population
C                E
DO -----> DA
Answer choice (A): This is the correct answer. The answer fails to shed any
light—positive or negative—on the connection between the ozone depletion
and the amphibian population decline. Because the argument is concerned with
the damage done by UV-B radiation, the fact that UV-B is the only damaging
type of radiation blocked by ozone is irrelevant.----就是说这种情况,只要是没有state directly about the relation of reason-effect, 那么就是irrelevant的。


Answer choice (D): This was the answer most frequently chosen by test takers.
This answer choice strengthens the argument by eliminating an alternate cause
for the effect (Type A). Had the natural habitat become smaller over the years
(from say, human encroachment or climatic change) then that shrinkage would
have offered an alternate explanation for the decline in the amphibian
population. By eliminating the possibility of habitat shrinkage, the stated cause
in the argument is strengthened.-----彻底没有想过,直接pass了,如果生活栖息地没有变小,证明A population不会因为这个原因而decline,那么就是否定了他因的存在从而加强了ozone layer depleted的原因。




之前遇到过的strengthen题目使用过这种eliminate alternative choice,那么Except题目也是可以使用的。


作者: xeyyxzty    时间: 2011-12-12 11:26
1. B:Amphibian populations are declining in numbers worldwide. Not coincidentally, the earth’s ozone layer has been continuously depleted throughout the last 50 years. P:Atmospheric ozone blocks UV-B, a type of ultraviolet radiation that is continuously produced by the sun, and which can damage genes. Because amphibians lack hair, hide, or feathers to shield them, they are particularly vulnerable to UV-B radiation. In addition, their gelatinous eggs lack the protection of leathery or hard shells.
C: Thus, the primary cause of the declining amphibian population is the depletion of the ozone layer.-->我要认真看题
is UV-B fatal to amphibian animals?
(A) Of the various types of radiation blocked by atmospheric ozone, UV-B is the only type that can damage genes.
--support--right
(B) Amphibian populations are declining far more rapidly than are the populations of nonamphibian species whose tissues and eggs have more natural protection from UV-B.
--support
(C) Atmospheric ozone has been significantly depleted above all the areas of the world in which amphibian populations are declining.
--support
(D) The natural habitat of amphibians has not become smaller over the past century.
--irrelevant--right
(E) Amphibian populations have declined continuously for the last 50 years.
--support
AD有点纠结~结果错了
Answer choice (A): This is the correct answer. The answer fails to shed any light—positive or negative—on the connection between the ozone depletion and the amphibian population decline. Because the argument is concerned with the damage done by UV-B radiation, the fact that UV-B is the only damaging type of radiation blocked by ozone is irrelevant.
Answer choice (D): This was the answer most frequently chosen by test takers. This answer choice strengthens the argument by eliminating an alternate cause for the effect (Type A). Had the natural habitat become smaller over the years (from say, human encroachment or climatic change) then that shrinkage would have offered an alternate explanation for the decline in the amphibian population. By eliminating the possibility of habitat shrinkage, the stated cause in the argument is strengthened.
2. B:At present the Hollywood Restaurant has only standard-height tables.
P: However, many customers come to watch the celebrities who frequent the Hollywood, and they would prefer tall tables with stools because such seating would afford a better view of the celebrities.  Moreover, diners seated on stools typically do not stay as long as diners seated at standard-height tables.
C: if the Hollywood replaced some of its seating with high tables and stools, its profits would increase.
costumers change their dishes
(A) some celebrities come to the Hollywood to be seen, and so might choose to sit at the tall tables if they were available
--???
(B) the price of meals ordered by celebrities dining at the Hollywood compensates for the longer time, if any, they spend lingering over their meals

--irrelevant
(C) a customer of the Hollywood who would choose to sit at a tall table would be an exception to the generalization about lingering

--irrelevant--right
(D) a restaurant's customers who spend less time at their meals typically order less expensive meals than those who remain at their meals longer

--right
(E) with enough tall tables to accommodate all the Hollywood's customers interested in such seating, there would be no view except of other tall tables

--irrelevant
又错了~~~好伤心3. B:The Calex Telecommunications Company is planning to introduce cellular telephone service into isolated coastal areas of Caladia, a move which will require considerable investment.
P:the only significant economic activity in these areas is small-scale coffee farming, and none of the coffee farmers make enough money to afford the monthly service fees that Calex would have to charge to make a profit.  +???
C: Calex contends that making the service available to these farmers will be profitable.
(A) Currently, Caladian coffee farmers are forced to sell their coffee to local buyers at whatever price those buyers choose to pay because the farmers are unable to remain in contact with outside buyers who generally offer higher prices.
--right
(B) In the coastal areas of Caladia where Calex proposes to introduce cellular telephone service, there is currently no fixed-line telephone service because fixed-line companies do not believe that they could recoup their investment.

--irrelevant
(C) A cellular telephone company can break even with a considerably smaller number of subscribers than a fixed-line company can, even in areas such as the Caladian coast, where there is no difficult terrain to drive up the costs of installing fixed lines.

--irrelevant
(D) Calex bases its monthly fees for cellular telephone service in a given region partly on the cost of installing the necessary equipment to provide the service there.

--irrelevant
(E) Calex has for years made a profit on cellular telephone service in Caladia's capital city, which is not far from the coastal region.

--irrelevant
4.B:With a record number of new companies starting up in Derderia and with previously established companies adding many jobs, a record number of new jobs were created last year in the Derderian economy.  
P:This year, previously established companies will not be adding as many new jobs overall as such companies added last year.
C:unless a record number of companies start up this year, Derderia will not break its record for new jobs created.
no new companies can offer more jobs
(A) Each year, new companies starting up create more new jobs overall than do previously established companies.
--opposite
(B) Companies established last year will not add a greater number of jobs overall this year than they did last year.

--the same with the premise
(C) This year, the new companies starting up will not provide substantially more jobs per company than did new companies last year.

--right
(D) This year, the overall number of jobs created by previously established companies will be less than the overall number of jobs lost at those companies.

--irrelevant
(E) The number of jobs created in the Derderian economy last year was substantially larger than the number of jobs lost last year.

--irrelevant
5.During the past year, Pro-Tect Insurance Company's total payout on car-theft claims has been larger than the company can afford to sustain.  ro-Tect cannot reduce the number of car-theft policies it carries, so it cannot protect itself against continued large payouts that way.  Therefore, Pro-Tect has decided to offer a discount to holders of car-theft policies whose cars have antitheft devices.  Many policyholders will respond to the discount by installing such devices, since the amount of the discount will within two years typically more than cover the cost of installation.  Thus, because cars with antitheft devices are rarely stolen, Pro-Tect's plan is likely to reduce its annual payouts.
In the argument above, the two portions in boldface play which of the following roles?BF在哪里???
作者: balapupu    时间: 2011-12-12 11:40
1.[34]
P: The number of A animal is decreasing+ the Ozone layer is depleting-?the ozone is blocking the B—> a chemical destroy genes-?because the A lack of hair and their eggs also lack protection.
C: the decline of A animal is because of the depleted Ozone layer.
Strength expect:
A:
(A) Of the various types of radiation blocked by
atmospheric ozone, UV-B is the only type
that can damage genes.-->support, remove other reasons.
(B) Amphibian populations are declining far more
rapidly than are the populations of
nonamphibian species whose tissues and
eggs have more natural protection from
UV-B.-->support, no cause , no effect.
(C) Atmospheric ozone has been significantly
depleted above all the areas of the world in
which amphibian populations are declining.-->support. Cause-effect.
(D) The natural habitat of amphibians has not
become smaller over the past century.-->irrelevant with the habit. R
(E) Amphibian populations have declined
continuously for the last 50 years.-->support, cause-effect.
作者: balapupu    时间: 2011-12-12 11:47
哎哎。。。我也做错了,话说最后排除是剩下了A和D,但是思路偏了,看着A更像是它因解释了。。。。囧。。
作者: 南瓜0729    时间: 2011-12-12 14:34
4.50s
Pzone blocks UV-B
p:UV-B harms A
c:the depletion of ozone--> the declining population

P:there is no other reason that leads to the declining population of A
A
A irrelevant
B bingo support UV-B is the cause
C bingo support the conclusion
D bingo
E bingo the depletion of ozone has happened in the last 50 years.

17.40s
P:many people perfer tall tables and they stay longer.
c:if they replace the high tables with short ones, the profits will increase.
P(weaken):this kind of people only came to the Hollywood Restaurant short period of time
E错了哎~~
看选项的时候没有抓住conclusion  


18.39s
P:introducing C phone to isolated area require considerable investment.
P:but famers don't have enough money and there is few economic activities there.
C:this action will be profitable.
P(support):in the long run, the company makes the profit.
A??  

排除法蒙对了  不太明白


19.45s
D 又错了~~查了半天 嘿嘿 弄明白了~


作者: phoebe0624    时间: 2011-12-12 22:51
【精练】
4.
P:there are no ozone layer to blocks UV-B
C:Amphibian populations are declining
Q: EXCEPT strengthen  
AA)
(A)无
(B) Strengthen  
UV-B is the reason that make the populations declining
(C)Strengthen  
Atmospheric ozone depleted makes the amphibian populations  declining.
(D) Strengthen  
to deny the other reason that lead to the  amphibians populations  declining
(E) irrevlent

【逻辑链】
17.
Q:weaken
P:replaced some of its seating with high tables and stools
C:the restaurant profits would increase
Ac)

(A) irrevent
not about the money
(B) S
trengthen  
the price of meals ordered by celebrities dining at the Hollywood compensates for the longer time, if any, they spend lingering over their meals
(C) irrevent
exception is irrevent about the profit
(D) S
trengthen  
long time more money
(E) Weaken
with enough tall tables  there would be no view except of other tall tables, so the profit would not increase.


18.
Q:support
P:
coffee farmers  make enough money to afford the monthly service fees
C:Calex contends that making the service available to these farmers will be profitable.
AC)
(A) irrevent
no contact may lead to farmers low profit
(B) irrevent
fixed-line companies is irrevent
(C) support
there is no difficult terrain to drive up the costs of installing fixed lines.
(D) irrevent
equipment
(E) irrevent
other region


19.
Q:Assumption
P:
copreviously established companies will not be adding as many new jobs overall as such companies added last year.  
C:Derderia will not break its record for new jobs created.
AC)
(A) weaken
Each year, new companies starting up create more new jobs overall than do previously established companies.
(B) repeat the stimuts
Companies established last year will not add a greater number of jobs overall this year than they did last year.
(C) assumptation
This year, the new companies starting up will not provide substantially more jobs per company than did new companies last year.
(D) irrevent
This year, the overall number of jobs created will be less than the overall number of jobs lost .
(E) weakn
The number of jobs created in the Derderian economy last year was substantially larger than the number of jobs lost last year.


20.
木有boldface During the past year, Pro-Tect Insurance Company's total payout on car-theft claims has been larger than the company can afford to sustain.  ro-Tect cannot reduce the number of car-theft policies it carries, so it cannot protect itself against continued large payouts that way.  Therefore, Pro-Tect has decided to offer a discount to holders of car-theft policies whose cars have antitheft devices.  Many policyholders will respond to the discount by installing such devices, since the amount of the discount will within two years typically more than cover the cost of installation.  Thus, because cars with antitheft devices are rarely stolen, Pro-Tect's plan is likely to reduce its annual payouts.

In the argument above, the two portions in boldface play which of the following roles?
udgment the argument uses in support of its conclusion about that outcome.
作者: bananazoo    时间: 2011-12-13 00:00
呀,发现最后一题,没有加粗……sorry~明天有电脑有电了就来加~囧………对了,大家17题是怎么做的?为什么是C呢?
作者: 南瓜0729    时间: 2011-12-13 11:54
不知道nana是哪个选项不明白 嘿 直接google吧 解析很详细
作者: winghyy    时间: 2011-12-13 12:24
精炼题
54s-strengthen EXCEPT
A population are declining in numbers. The ozone layer has been depleted and the ozone blocks UVB. Because A lacks hair, hide or feathers to shield UVB. So the primary cause of the declining amphibian population
is the depletion of the ozone layer.
选D  【正确答案A】
(A) Of the various types of radiation blocked by atmospheric ozone, UV-B is the only type that can damage genes.  
——strengthen [结论涉及的两个元素是 the ozone depletion和the amphibian population decline,与UVB无关]
(B) Amphibian populations are declining far more rapidly than are the populations of
nonamphibian species whose tissues and eggs have more natural protection from UV-B.  ————strengthen
(C) Atmospheric ozone has been significantly depleted above all the areas of the world in
which amphibian populations are declining. ——strengthen
(D) The natural habitat of amphibians has not become smaller over the past century. ——irrelevant
(E) Amphibian populations have declined continuously for the last 50 years.——strengthen


Answer choice (A): This is the correct answer. The answer fails to shed any
light—positive or negative—on the connection between the ozone depletion
and the amphibian population decline.
Because the argument is concerned with
the damage done by UV-B radiation, the fact that UV-B is the only damaging
type of radiation blocked by ozone is irrelevant.

Answer choice (D): This was the answer most frequently chosen by test takers.
This answer choice strengthens the argument by eliminating an alternate cause
for the effect (Type A).
Had the natural habitat become smaller over the years
(from say, human encroachment or climatic change) then that shrinkage would
have offered an alternate explanation for the decline in the amphibian
population. By eliminating the possibility of habitat shrinkage, the stated cause
in the argument is strengthened.

作者: 风无衣    时间: 2011-12-13 14:46
【精炼2-3】
strengthen X_55s
background information: Amphibian populations are declining and the ozone layer has been continuously depleted throughout the last 50 years.ozone blocks UV-B,which can damage genes.
premise:amphibian popultions lack hide from UV-B and their eggs also lack protection
conclusion:the depletion of ozone layer is the reason for the declining amphibian popution.
(A) Of the various types of radiation blocked by
atmospheric ozone, UV-B is the only type
that can damage genes.
(B) Amphibian populations are declining far more
rapidly than are the populations of
nonamphibian species whose tissues and
eggs have more natural protection from
UV-B.——support
(C) Atmospheric ozone has been significantly
depleted above all the areas of the world in
which amphibian populations are declining.——support
(D) The natural habitat of amphibians has not
become smaller over the past century.
(E) Amphibian populations have declined
continuously for the last 50 years.——support,match the time of ozone depletion.

额~我昨天做了LSAT,刚刚好做到了这道一模一样的题目,昨天做的时候选的A。结果答案说选错了!!!结果今天在这一看,答案选A!!!晕啊!!!~~~~(>_<)~~~~ 到底选什么,这道题目!!!求解啊!!!
好,先看看帖子里面贴的答案~
Answer choice (A): This is the correct answer. The answer fails to shed any
light—positive or negative—on the connection between the ozone depletion
and the amphibian population decline. Because the argument is concerned with
the damage done by UV-B radiation, the fact that UV-B is the only damaging
type of radiation blocked by ozone is irrelevant.
Answer choice (D): This was the answer most frequently chosen by test takers.
This answer choice strengthens the argument by eliminating an alternate cause
for the effect (Type A). Had the natural habitat become smaller over the years
(from say, human encroachment or climatic change) then that shrinkage would
have offered an alternate explanation for the decline in the amphibian
population.
By eliminating the possibility of habitat shrinkage, the stated cause
in the argument is strengthened.

好吧~再看看LSAT的答案~
答案:D
思路:原文论述臭氧层的破坏造成UV-B辐射过量是导致两栖动物数量减少的主要原因。
A:支持说明为什么是UV-B而不是其他原因。
B:支持说明为什么UV-B会降低两栖动物的数量。
C:提供空间论据说明臭氧层受损和两栖动物数量减少的相关性。
D:中性评价。不Weaken也不Strengthen.
E:提供时间论据说明臭氧层受损和两栖动物数量减少的相关性。


某NN说的技巧:做EXCEPT题如果答案中出现原文没有的专有名词(地名和人名等)一般就选它。

好吧~看了老半天还是比较支持帖子里面的答案~哎~~~+_+
因为结论是the primary cause of the declining amphibian population is the depletion of the ozone layer.我感觉LSAT给的思路自我延伸了,原结论只说臭氧层的破坏是amphibian人口减少的主要原因,关注的是UV-B带来的damage,是不是唯一的type应该无关~

作者: winghyy    时间: 2011-12-13 18:05
逻辑链
17. 50s-weaken
Many customers come to watch the celebrities, and they would prefer tall tables for a better view of the celebrities.  Moreover, diners seated on stools typically do not stay as long as diners seated at standard-height tables.  Therefore, if the Hollywood replaced some of its seating with high tables and stools, its profits would increase.
Prephase: In order to watch celebirites, they will spend less.
选D

18. The C telecommunication company plans to introduce celluar telephone service to isolate costal areas. This plan requires a big investiment.  The small-scale coffee farming is the only major economic acitivities. The coffee farmers can not offer the charge. Nevertheless, Calex contends that making the service available to these farmers will be profitable.
选A

19. 52s-assumption
Last year, with a record number of new companies starting up and with previously establised companies adding many jobs, a record number of new jobs created last year.
Establised companies will not add as many new jobs this year as last year. Therefore, unless a record number of companies start up this year, it will not break its record for the new jobs created.
选B

20. 53s-bold face
P insurance company cannot afford the payout. Because P cannot reduce the number of car-theft policies it carries, it cannot protect itself against continued large patyout. P has decided to offer a discount to holders of car-theft policies whose cars have antitheft devices. The amount of discount will cover the cost of installation within 2 years. Therefore, because cars with antitheft devices are rarely stolen, P's plan is likely to reduce its annual payout.
Prephase: 1st premise   2st Conculusion
选E

今天错得有点夸张了。。。

作者: zz42050524    时间: 2011-12-13 18:51
2011/12/13 support expect 41s
The population of amphibian is declining. The ozone layer is destroyed either letting ultraviolet radiation run through atmosphere. Because the amphibian lacks hair and shell,
The declining of amphibian is caused by decreasing of ozone layer.
Pre: 1.没有其他原因导致海豚减少
2. 臭氧层未被严重破坏地区减少缓慢
A) Of the various types of radiation blocked by    只是说UV-B是能破坏基因,和结论的的内
atmospheric ozone, UV-B is the only type          无关,这个是答案。
that can damage genes.
(B) Amphibian populations are declining far more   support
rapidly than are the populations of
nonamphibian species whose tissues and
eggs have more natural protection from UV-B.
(C) Atmospheric ozone has been significantly    support
depleted above all the areas of the world in
which amphibian populations are declining.
(D) The natural habitat of amphibians has not    support 排除其他原因
become smaller over the past century.
(E) Amphibian populations have declined     support
continuously for the last 50 years.
作者: ugly5552000    时间: 2011-12-18 18:04
1/Background: amphibian population are declining. The UV-B radiation emitted by the sun is harmful to amphibians. The atmospheric ozone blocks the UV-B radiation. The earth’s ozone layer has been continuously depleted.
Premise: the decline of amphibian populations is due to the depletion of ozone layer.
Prephrase: something has nothing to do with ozone layer.
A
A: shows no connection with the argument
B: the depletion of ozone layer lead more amphibians be radiated by the UV-B
C: the depletion of ozone lay is consistent with the decline of amphibians.
D: pay attention to “over the past century”, means before the ozone layer was depleted, the amphibians did not decline, but now, they decline.
E: strengthen
2/Background: because of the view advantage, many customers prefer tall tables. The Hollywood restaurant has only standard-height tables.
Premise: since diners seated on stools typically stay shorter than diners seated at standard-height tables, the restaurants will earn more profit if they supplant all the standard-height tables with tall tables.
Prephrase: the customers seated on stools often orders much less dishes than those seated at standard-height tables.
C
A: irrelevant
B: irrelevant
C: since customers who sit at the tall tables will not linger, the seated time will be long, and the conclusion cannot stand.
D: expensive meals do not guarantee more revenue.
E: irrelevant
3/Background: a telephone company plans to introduce telephones service into isolated coastal areas of Caladia, but the farmers, the main economic activists in the areas, cannot afford the price of the service.
Premise: nevertheless, the company still believe the service in the areas will be profitable.
Prephrase: the telephone company can set up commercial connects between the farmers and the coffee market, which will increase the farmers’ revenue considerably.
A
A: the telephone company can make the farmers to contact with outside buyers, who will increase the farmers revenue, then the farmers can afford the service
B: cannot explain the company will success in the areas.
C: the farmers may even be unable to pay the fee which is lower than the fixed telephone.
D: irrelevant
E: irrelevant
4/Background: with a record number of new companies setting up in D and with previously established companies adding many jobs, a record number of new jobs were created last year. This years, the jobs provided by the previously established companies are not as many as those in the last year, the jobs creating record cannot be broken.
Premise: unless a record number of companies start up this year, D will not break the record of new jobs created.
Premise: the companies established last year cannot add more new jobs than they did last year.
C
A: since the new jobs from the new companies will increase, the total new jobs may more than last year.
B: this is the information
C: the new companies still has less new jobs to add, the total will be less
D: irrelevant
E: irrelevant
5/Background: Pro-tect company cannot afford the total payout on car-theft caims.
Premise: Pro-tect plans to provide a discount to enable the car holders to install anti-theft devices on their cars. As the discount is more than the cost of the device, many car holders will respond to the discount, and since the cars with antitheft devices are much less likely to be stolen than those without antitheft devices, Pro-tect’ s plan is likely to reduce its annuel payout.
B
A: not both evidence
B: right
C: not position
D: not explaination
E: not outcome
作者: leewonting    时间: 2012-4-20 19:24
103
At present the Hollywood Restaurant has only standard-height tables.  However, many customers come to watch the celebrities who frequent the Hollywood, and they would prefer tall tables with stools because such seating would afford a better view of the celebrities.  Moreover, diners seated on stools typically do not stay as long as diners seated at standard-height tables.  Therefore, if the Hollywood replaced some of its seating with high tables and stools, its profits would increase.
The argument is vulnerable to criticism on the grounds that it gives reason to believe that it is likely that
(A) some celebrities come to the Hollywood to be seen, and so might choose to sit at the tall tables if they were available
(B) the price of meals ordered by celebrities dining at the Hollywood compensates for the longer time, if any, they spend lingering over their meals
(C) a customer of the Hollywood who would choose to sit at a tall table would be an exception to the generalization about lingering
(D) a restaurant's customers who spend less time at their meals typically order less expensive meals than those who remain at their meals longer
(E) with enough tall tables to accommodate all the Hollywood's customers interested in such seating, there would be no view except of other tall tables
背景:At present the Hollywood Restaurant has only standard-height tables.
条件:Many customers come to watch the celebrities who frequent the Hollywood and they would prefer tall tables with stools because such seating would afford a better view of celebrities and diners on stools typically do not as long as diners seated at standard-height tables  
结论:It’s profitable to replace the standard-height with stools.
推测:停留在餐厅的时间越长,消费越高
选D
选C,其实看celebrities的人只是大多数人的一点点。而D有一个gap,可能停留时间长的人会order更贵的东西,但是如果停留时间短的人比停留长的人更多,那么即使order比较便宜也会比较多profit


104
背景:The Calex Telecommunications Company is planning to introduce cellular telephone service into isolated coastal areas of Caladia where has only small-scale coffee farming
条件:The farmers earn too little to pay the service fees
结论:But the company insists to introduce the plan
推测:farmers有了电话,生意好了!
选A


105
背景:There is a record number of new companies starting up in Derderia and with previously established companies adding many jobs.
条件:Previously established companies will not be adding as many as new jobs as much companies added last years
结论:Therefore, unless a record number of companies start up this year, Derderia will not break its record for new jobs created.
推测:record= new starting companies + previously established companies offering new jobs, 如果以前的公司没有更多的新工作,那么如果新公司没有更多,那么new jobs不能record
选C

106
背景:During the past year, Pro-Tech Insurance Company’s total payout on car-theft claims has been larger than the company can afford to sustain. Pro-Tech can not reduce the number of car-theft policies it carries, so it cannot protect itself against continued large payouts that way
条件:Pro-Tech has decided to offer a discount to holders of car-theft policies whose cars have antitheft devices.
结论:Thus, because cars with antitheft devices are rarely stolen, Pro-Tech’s plan is likely to reduce its annual payouts
推测:第一部分是situation?第二部分应该是结果?
选B
作者: baseboss    时间: 2012-6-5 13:19
cellular telephone service, there is currently no fixed-line telephone service because fixed-line companies do not believe that they could recoup their investment.
-->irrelevant
(C) A cellular telephone company can break even with a considerably smaller number of subscribers than a fixed-line company can, even in areas such as the Caladian coast, where there is no difficult terrain to drive up the costs of installing fixed lines.
-->irrelevant
(D) Calex bases its monthly fees for cellular telephone service in a given region partly on the cost of installing the necessary equipment to provide the service there.
-->irrelevant
(E) Calex has for years made a profit on cellular telephone service in Caladia's capital city, which is not far from the coastal region.
-->irrelevant

4 presume:
The previously new company will not add new jobs
A) Each year, new companies starting up create more new jobs overall than do previously established companies.
-->irrelevant
(B) Companies established last year will not add a greater number of jobs overall this year than they did last year.
-->corrct
(C) This year, the new companies starting up will not provide substantially more jobs per company than did new companies last year.
-->irrelevant
(D) This year, the overall number of jobs created by previously established companies will be less than the overall number of jobs lost at those companies.
-->irrelevant
(E) The number of jobs created in the Derderian economy last year was substantially larger than the number of jobs lost last year.
-->irrelevant

5
1.evidence and presume for conclusion
2conclusion

(A) The first and the second are both evidence offered by the argument as support for its main conclusion.
-->incorrect
(B) The first presents a problem a response to which the argument assesses; the second is the judgment reached by that assessment.
-->incorrect
(C) The first is the position the argument seeks to establish; the second is a judgment the argument uses to support that position.
-->incorrect
(D) The first is a development that the argument seeks to explain; the second is a prediction the argument makes in support of the explanation it offers.
-->incorrect
(E) The first presents a development whose likely outcome is at issue in the argument; the second is a judgment
-->correct
作者: jetyxo    时间: 2012-6-26 17:22
1 31":  ppulation of A declined ,at the same time, ozone layers depleted ---> UV-b, broken genes
           A has no hair,feather and A's egg lack hard shells---> A is vulnerable to UV-b
        conclusion: the depletion of ozone layer leads to decline of population of A
        strengthen EXCEPT? B C E support D: no other reason leads to decline ,also supports
        answer:A
---------------------------------------------------------------------
2 35" premise:people like high tables and stools to look celebrity;
                      diner at high tables do not  stay as longer as diner at standard tables
        conclusion:  to make more profits ,the restaurant should replace the srandard tables with high tables
       illogic reason because? may be diner at high tables consume less than at standard tables
          answer

  做错了: D正好是支持,看反了。。。  answer:C
------------------------------------------------------------
3 29" background: CT plan to give service to C, this plan need huge investment;
                             farmer in C,can not aford the service
           however: CT still decide to invest in C, because it think it will be profitable.
          reason? A
-----------------------------------------------------
4 26" last year, a new record created by two things: A a large employment in old companies
                                                                                B a large number of new opend companies
          this year, no A,so in order to make a new record, must need a more huge number of new opend companies
         weaken:  just few opening companies, but each of them employ more and more people
        answer:C
--------------------------------------------------
5  B
作者: meabh    时间: 2012-7-12 07:50
4. (1) Time: 31s
(2) Category: Weaken
(3) Logical pattern:
Background: A is declining.
Premise: Ozone layer depleted to prevent U and A is vulnerable to U.
Conclusion: U declined A.
(4) Prephrase: When ozone is not depleted A’s number doesn’t decline.
(5) Analysis:
(A) Of the various types of radiation blocked by atmospheric ozone, UV-B is the only type that can damage genes.---irrelevant. Genes vs number
(B) Amphibian populations are declining far more rapidly than are the populations of nonamphibian species whose tissues and eggs have more natural protection from UV-B.----Strengthen
(C) Atmospheric ozone has been significantly depleted above all the areas of the world in which amphibian populations are declining.-----Strengthen
(D) The natural habitat of amphibians has not become smaller over the past century.----strengthen,他因削弱
(E) Amphibian populations have declined continuously for the last 50 years.-----Strengthen

17. (1) Time: 42s
(2) Category: weaken
(3) Logical pattern:
Background: people eat in H to see stars.
Premise: people don’t stay long time in high chairs.
Conclusion: high tables will increase profit.
(4) Prephrase: people here to see stars prefer high tables and won’t leave quickly.
(5) Analysis: C

18. (1) Time: 31s
(2) Category: Strengthen
(3) Logical pattern:
Background: C will invest in Canada.
Premise: farmers have less money to afford the service.
Conclusion: service to farmers will profit.
(4) Prephrase: service brings profit to farmers so they are willing to do so.
(5) Analysis: A

19. (1) Time: 36s
(2) Category: assumption
(3) Logical pattern:
Background: last year a record of new companies created many jobs.
Premise: previously companies don’t create more jobs.
Conclusion: jobs will not be created as many as before unless more new companies opened.
(4) Prephrase: The newly existed companies don’t create more jobs.
(5) Analysis: C

20. (1) Time: 45s
(2) Category: Boldface
(3) Logical pattern:
Background: P cannot reduce the payouts because the thefts cannot reduce.
Premise: P offered discount for antitheft.
Conclusion: P will reduce the payouts.
(4) Prephrase:
(5) Analysis: B
作者: Donts    时间: 2012-7-22 10:59
1.计时49”
premise:the ozone which can protect genes and block uv-b has been continuously depleted; the A is lack of hair,shell and leather to protect itself; the eggs are vulnerable.
conclusion:the depletion of the ozone is the primary cause of A decrease.
(A) Of the various types of radiation blocked by atmospheric ozone, UV-B is the only type
that can damage genes.加强
(B) Amphibian populations are declining far more rapidly than are the populations of nonamphibian species whose tissues and eggs have more natural protection from UV-B.加强
(C) Atmospheric ozone has been significantly depleted above all the areas of the world in
which amphibian populations are declining.加强
(D) The natural habitat of amphibians has not become smaller over the past century.排除它因,加强
(E) Amphibian populations have declined continuously for the last 50 years.无关
选择E
2.计时31''
premise: customers like to sit on stools to watch celebrities; customers sit on stools stay shorter than those sit on standard seats.
conclusion: replace standard seats with stools can increase profits.
选择C
3.计时45''
calex plans to invest service in a place with the only economic activity---coffee farming; the farmers make inadequate money to affort to the fees that would make calex reap a profit.
选择A
4.计时50''
premise: newly openly companies created many jobs with previously established companies. this year, previously established companies create less job opportunities than last year.
conclusion: if D wants to break the record for new jobs created, then it has to open a number of new companies.
推测:选择C
5.计时45''
premise:insurance company decide to offer a discount to car owners who install antitheft devices on their cars. the car owners will be attracted to install devices on their cars.

conclusion:the company can reduce its annul payout because of the security devices installed.
选择E
订正
1.A。。。。
没看到E里的last 50 years
A只有ozone,没有Am.
5.B
总结
unless改写。
作者: wanggang0411    时间: 2012-8-31 16:18
52
Premise: The depletion of atmosphere.
Conclusion: amphibian populations are decling by the reason of the depletion.
Execpt quesion: A answer choice which doesn't support the conclusion.
A:

1:20
Premise: diners seated on the stools do not stay as long as the diners seated at standard-height tables.
Conclusion: Replace some of its seating with high tables and stools will increase profits.
weaken Inference:
The author falsely rests on a gratuitous assumption that replacing some seats will attract more customers.
However, this is not necessarily the case, since no evidence has been stated in the argument to support this assumption.
In fact, it is possible that people who watch the celeberaties are more prefer to stay in a restaurant in which
everybody come here to watch celeberaties. Thurs, this argument is unwarranted untill such possbilities have been concerned and ruled out.

This argument also rests on another insufficient evidence that because diners seated on the stools will not stay as long as the diners seated on the standard tables
to draw a conclusion that the resturarant will accommodate more customers.
But, only one example is logically unsounded to establish a general concluision.
It is possible that customers who seat on the stools will stay more longer after diner to watch celebration.
So, on the ground of such limited evidence, the conclusion is completely unwarranted.
D
C
The most important, this question is not to ask which answer choice weakens the argument, but actually the question is to ask which answer choice is an assumption based on which the conclusion have been made.
It's quite tricky:p

35
Premise: To introduce celluar telephone, company need to do investment.
Premise: Coffee farming is the only significant activity, but the farmer can't afford the telephone fees.
Conclusion: The company can still gain profit through the introduction of celluar telephone.
Support: by some reason, farmers or other social class will afford the fee after the introduction.
A: same as the inference.
A

28
Premise: The existing company will not offer more jobs this year than last year.
Conclsuion: Unless there are more new companies this year than last year, the number of new jobs in this year will lower than that in last year.
Assumption: Each of new start up company this year can offer no more jobs than did the each of start up company last year
C is same as the inference.
C

1:09
The second is a conclusion, the first is an issue to be resolved by a premise to support the conclusion.
B: The first presents a problem a response to which the argument assesses, the second is the judgement reached by that assessment.
B
作者: srafcatt    时间: 2012-10-23 22:19
精炼
weaken 42s
amphibian NO.declines. ozone layer deplete
ozone layer block UV-B from damaging genes.--hurt amphibians & eggs
conclusionzone layer-->declining amphibian population
ozone layer also hurt the predator of amphibian
answer
A confirm the information that UV-B can damage genes.
B confirm the information mentioned in the question stem
C strenghten
D weaken
E strenghten
--------------------------------------------------------------------------
逻辑链:
1.weaken 28s
premise:standard-height tables sit longer,and tall tables with stools have a better view of celebrity.
conclusion:the profit will increase
the customer will leave more frequently so that the new customers will come in.
answer
A irrelevant information
B strenghten
C strenghten
D weaken
E irrelevant
2.strenghten  30s
 background: tele co. invest cellular service
 premise: farmers poor coffee farming
 conclusion: co. make profits
co. service could bring more profits to farmers so that they can afford it.
answer:A
A correct answer
B this information cannot assure the co.this service will gain profits.
C irrelevant information
D irrelevant information
E irrelevant information
3.assumption 22s
the number of jobs increased.
this yr will not break the record
only these two way to add jobs
answer:B
A irrelevant
B correct
C irrelevant
D irrelevant
E irrelevant
4.boldface 30s
在过去的一年,“保护”保险公司在汽车偷盗上的全部支出已经超过了公司可以持续支付的数额。“保护”不能减少它的汽车偷盗保险单,所以它不能在如此大量的支付下保护自己。因此,“保护”已经决定为装有防贼装置的车主提供保险购买折扣。许多投保人将通过装防盗装置相应这个政策,因为两年后保险折扣省下的钱远高于装置的费用。因此,因为有防贼装置的车很少被偷,“保护”的计划很可能会减少它每年的支出。
选B
作者: srafcatt    时间: 2012-10-23 22:48
今天的题可真难哪
纠错:
精练题
选A 的原因是题干中谈的是紫外线造成的破坏--基因断裂,而选项谈的是只有紫外线会造成杀伤力,这是irrelevant的。
不选D 是因为D 证实了两栖类的减少不是因为栖息地的缩小,这叫排他因。

逻辑链的第一题是“true type”类型的题,这种题选项不能引入新的信息。

must be true 介绍
Must be true的特征由提面推断选项,题目是支持选项的,选项是题面的结论。题面的信息都毋庸置疑(如果must be true的题目说太阳打西边出来的,你也绝对不能怀疑它的可能性)。尤其是要注意一下表示限定的词many, some, none, must, entirely等,每一个限定范围的形容词都很重要,因此要仔细阅读题目和题干。
  MBT的正确答案:永远都可以被题面中的信息证明。题面信息所涉及的范围经常能帮你排除1个以上的错误选项。
  MBT题的特征:没有结论。
  Must be true正确选项的标志:
  1、原文改写or复述原文,复述了一部分原文中的信息,只是表达方式不同。
  2、2个或者2个以上的前提结合产生的东西,是原文中几条信息的组合。
  Must be true错误选项的标志:
  1、可能是true,也可能不是。
  选项一定要是你读到的内容的must be true的内容不能带入自己的想象力,如果稍微有点可能但是又不是一定会发生的事一般都是错的
  2、扩大原文的范围,也是典型的错误答案。
  放大了原文的信息,比原文提供的信息内容要宽泛。尤其当选项复述原文,但同时改变了原文范围的时候,就是典型的陷阱。可以和上面的限定词结合,比如原文说的是some people,但是选项变成了most people那就不对了;
  3、有原文中没有提到的新信息出现
  must be true是尽量避免提到新鲜的事情,几个前提推出结论的就不算新事物啦。
  怎么检验是否是新信息:
  (1)新信息不在原文的概念范围内。
  (2)新信息不是由原文的条件组合后产生的。
  4、偷换概念
  一个原文中提到过的概念,被选项中的另一个很相似的概念偷换了。
  这是gmac的老倌们最爱玩的把戏,比如题目说某个特定的东西A怎么怎么样,但是选项里面也提到了A,但是如果没有特定的修饰成为,A可能就变成了泛指而不是题目特质
  5、因果倒置
  这类型的话一定要仔细,原文说A可以推出B,答案可能会变成B推出了A。(因果倒置的题目的否定情况好像是可以出现在support题里面的,因为它消除了因果倒置的理由)
  6、颠倒次序
  非常容易迷惑人,是将原文中的A of B换成了B of A的选项,改变了原文信息出现的位置和他们的逻辑链以及表达的意思。
  如:Many people have some type of security system in their home.
  选项成了:some people have many types of security in their home.

本题D错的原因就是引入了新的信息。



逻辑链第三题:
新公司和老公司去年都增加新的人员,导致这个地区的新的工作机会总体增加。

今年老公司没有增加员工,除非出现大量的新公司,否则今年增长的就业机会不会超过去年的记录。


问题目是基于什么假设

关于是否能突破记录有2条,

1 新增加的公司数目多于去年,但是每家公司的平均雇佣人数和去年一样

2 今年新开公司数目不多于,但是每家新公司雇佣的人员是大大多于去年平均每家新公司的雇佣人数的



答案 C 说的是1,基于每家新公司雇佣人数不多于去年这个假设,如果今年没有更多的新公司开,则新的就业机会不会突破去年的记录

A 新的公司雇佣人数是多于老公司雇佣人数的,此题是新老公司雇佣人数去年和今年的比较,A是新老公司之间比较 杀

我只能说,我做题做糊涂了。完全读错。阅读不过关......铁板神猴呀,是时候去你那里逛逛了~
作者: oytt1111    时间: 2012-10-24 17:12
4.Time:59s
Premise: A populations are declining
Premise: ozone layer which blocks UVB has been depleted. UVB damage A.
Premise: A are vulnerable due to eggs and no hair
Conclusion: depletion of ozone layer is primary cause of declining A population
Type: strengthenX
Answer: D
D is irrelevant while others are strengthen.

17. Time: 56s
Premise: HR has only standard tables.
Premise: celebrities viewers prefer tall to standard.
Premise: diners on tall tables stay shorter than on standard.
Conclusion: If HR replace some to tall, profits will increase.
Type: weaken
Answer: D

18. Time: 61s
Premise: CTC is to introduce cts to C, and will spend a big money
Premise: farmers in C will not have enough money to pay CTC
Conclusion: CTC contends that the program will be profitable.
Type: strengthen
Answer: A

19. Time: 36s
Premise: a record number of new jobs were created last year because two reasons, new companies brought new jobs and old companies added new jobs.
Premise: old jobs will not add as many as last years.
Conclusion: unless a great number of new companies start up, it will not break the record.
Type: assumption
Answer: B

A1 + B1 < C1,
B2< B1 and  ?
C2<C1


20. Time: 80s
Premise: PTIC payout was larger than they can afford.
Premise: PTIC cannot change the number of payout then they offer a discount to holders with a device.
Premise: more customers will buy device to get the discount.
Conclusion: PTIC will reduce its annual payouts.
Type: evaluation
Answer: B
作者: jinyueyue    时间: 2012-10-25 08:59
10.23+【2-3】
4.48s
BG:a population declining
PREzone layer depleted uv increased;
       a are vulnerable to uv
CONzone depetion cause a decline
可能方向:有其他原因造成a减少 我选择D 正选A
错误分析:A为非正非负态度,所以不可以加强;D加强了结论
17.31s
BG:目前h餐厅只有标准桌
PRE:高桌视野好;
       stools吃得快
CON:h如果换高桌和stools利润会增加
可能方向:换了利润不会增加 我选择C
18.20s
BG:ct公司要以高投资向c区域引入移动电话
PRE:c地区唯一经济来源是小规模咖啡业;
       当地农民付不起移动电话服务费
CON:ct公司会赔钱
可能方向:有个原因ct不赔钱 我选择:A
19.25s
BG:去年d公司新增了很多new jobs(新公司建立+旧公司增加岗位)
PRE:今年旧公司不会增加和去年一样多的岗位
CON:除非新公司建立,否则D就打不破今年new jobs created的记录
可能方向:只有这两个原因 我选择B 正选C
错误分析:没看清选项 B知识重复了PRE!错!
10.53s
BG:pt保险公司去年的丢车赔偿超过了他能承受的范围
PRE:pt减少不了丢车,只好在赔偿上想办法
CON:装了antitheft devices的车主 丢车记录减少 pt赔偿费用减少
我选择:E 正选B
错误分析:B比E准确 present在此处为“提出”的意思
作者: ariel0923    时间: 2012-10-26 19:07
(A) Of the various types of radiation blocked by
atmospheric ozone, UV-B is the only type
that can damage genes.
(B) Amphibian populations are declining far more
rapidly than are the populations of
nonamphibian species whose tissues and
eggs have more natural protection from
UV-B.
(C) Atmospheric ozone has been significantly
depleted above all the areas of the world in
which amphibian populations are declining.
(D) The natural habitat of amphibians has not
become smaller over the past century.
(E) Amphibian populations have declined
continuously for the last 50 years.

精练错选D
A:错在不管是不是只有UVB伤害基因,都是大气层的关系,所以这个既没有加强也没有削弱,无关选项
作者: zxppx    时间: 2013-1-18 10:09
看来今天的精炼很难啊!
4.

Amphibian populations are declining in numbers
worldwide. Not coincidentally, the earth’s ozone
layer has been continuously depleted throughout the
last 50 years. Atmospheric ozone blocks UV-B, a
type of ultraviolet radiation that is continuously
produced by the sun, and which can damage genes.
Because amphibians lack hair, hide, or feathers to
shield them, they are particularly vulnerable to UV-B
radiation. In addition, their gelatinous eggs lack the
protection of leathery or hard shells. Thus, the
primary cause of the declining amphibian population
is the depletion of the ozone layer.

The logic chain of the argument is that because of decline in the population of amphibians and in the amount of ozone, the latter caused the former. What’s more, UV-B can harm amphibians with the depletion of the ozone layer. In order to solve the argument, we need to find the right answer, which cannot strengthen the causality provided in the argument.
Each of the following, if true, would strengthen the
argument EXCEPT:

(A) Of the various types of radiation blocked by
atmospheric ozone, UV-B is the only type
that can damage genes.

At first glance, it seems to be irrelevant. But a more thorough analysis reveals that it is useful for the argument. If other factors can reduce the population of amphibians, the reasoning referred in the argument will be weakened. 但问题是即使有其他的射线可以造成伤害,请注意选项中的限定词“被臭氧层所阻挡的”,如果臭氧层不被破坏,即使有其他的射线,也不能造成伤害,所以这个选项并没有加强结论!

But please notice the conclusion, which refers to the amphibian population and the ozone layer, has nothing to do with the UV-B, even though UV-B can harm amphibians provided by the argument. So whether UV-B is the only type of the various types of radiation blocked by atmospheric ozone can damage genes has no effect on the argument.
(B) Amphibian populations are declining far more
rapidly than are the populations of
nonamphibian species whose tissues and
eggs have more natural protection from
UV-B.

Actually, this answer choice strengthens the argument.

This choice strengthens the argument by showing that when the cause in the nonamphibian speices is absent, the effect does not occur.
(C) Atmospheric ozone has been significantly
depleted above all the areas of the world in
which amphibian populations are declining.

The same to B.

This strengthens the argument by showing that when the cause occurs, the effect occurs, too.
(D) The natural habitat of amphibians has not
become smaller over the past century.

In this case, we can find out that if the natural habitat of amphibians has not become smaller over the past century, and that if the earth’s ozone layer has been continuously depleted throughout the past half century, these two seems to irrelevant. BA

事实上,如果栖息地减少也会造成该物种的数量减少,排除他因,加强结论

This answer strengthens the argument by eliminating an alternative cause for the effect. Had the natural habitat of amphibians become smaller over the years (from say, human encroachment or climatic change) then that shrinkage would have offered an alternative explanation for the decline in the population of amphibians.
(E) Amphibian populations have declined
continuously for the last 50 years.

The same to B.

This answer choice strengthens the argument by showing that the decline in amphibians has mirrored the depletion of ozone layer, thus affirming the data used to make the conclusion.

17.
At present the Hollywood Restaurant has only standard-height tables. However, many customers come to watch the celebrities who frequent the Hollywood, and they would prefer tall tables with stools because such seating would afford a better view of the celebrities. Moreover, diners seated on stools typically do not stay as long as diners seated at standard-height tables. Therefore, if the Hollywood replaced some of its seating with high tables and stools, its profits would increase.
The argument is vulnerable to criticism on the grounds that it gives reason to believe that it is likely that

(A) some celebrities come to the Hollywood to be seen, and so might choose to sit at the tall tables if they were available

This answer choice strengthens the argument by showing that tall tables are more popular than standard-height.
(B) the price of meals ordered by celebrities dining at the Hollywood compensates for the longer time, if any, they spend lingering over their meals

The argument has pointed out that many customers prefer tall tables with stools. If they can linger longer time, the price of meals can compensates for the time.
(C) a customer of the Hollywood who would choose to sit at a tall table would be an exception to the generalization about lingering

“Exception to the generalization about lingering” means these customers would stay very long time, thus reducing the total customers in the restaurant and weakening the argument. BA

the 'generalization about lingering' is the idea that people won't sit at tall tables for as long as at short tables. Yet, according to the passage, the customers at tall tables will probably be there to gawk at celebrities, so they will probably stay for a long time - the 'exception' mentioned in this answer choice.From Manhattan
(D) a restaurant's customers who spend less time at their meals typically order less expensive meals than those who remain at their meals longer

This answer choice supports the high tables and stools.
(E) with enough tall tables to accommodate all the Hollywood's customers interested in such seating, there would be no view except of other tall tables

I do not understand the latter part in the sentence. In fact, it does not refer to the lingering time of customers, thus not weakening the argument.



18.
The Calex Telecommunications Company is planning to introduce cellular telephone service into isolated coastal areas of Caladia, a move which will require considerable investment. However, the only significant economic activity in these areas is small-scale coffee farming, and none of the coffee farmers make enough money to afford the monthly service fees that Calex would have to charge to make a profit. Nevertheless, Calex contends that making the service available to these farmers will be profitable.
Which of the following, if true, provides the strongest support for Calex's contention?

(A) Currently, Caladian coffee farmers are forced to sell their coffee to local buyers at whatever price those buyers choose to pay because the farmers are unable to remain in contact with outside buyers who generally offer higher prices.

In this case, the answer choice strengthens Calex’ contention by showing that if it can help those farmers to contact with outside buyers who generally offer higher prices, the revenues of the farmers will increase, and they can pay the fees. BA
(B) In the coastal areas of Caladia where Calex proposes to introduce cellular telephone service, there is currently no fixed-line telephone service because fixed-line companies do not believe that they could recoup their investment.

Actually, this answer choice weakens the argument.
(C) A cellular telephone company can break even with a considerably smaller number of subscribers than a fixed-line company can, even in areas such as the Caladian coast, where there is no difficult terrain to drive up the costs of installing fixed lines.

The comparison between a cellular telephone and a fixed-line company is irrelevant.
(D) Calex bases its monthly fees for cellular telephone service in a given region partly on the cost of installing the necessary equipment to provide the service there.

Since we have already known that the cost of installing is considerable, and these farmers cannot afford the monthly fees, this answer choice cannot strengthen the argument.
(E) Calex has for years made a profit on cellular telephone service in Caladia's capital city, which is not far from the coastal region.

Since the conditions between the two places are different, we cannot make sure Calex will make a profit on cellular telephone service in the coastal region.



19.
With a record number of new companies starting up in Derderia and with previously established companies adding many jobs, a record number of new jobs were created last year in the Derderian economy. This year, previously established companies will not be adding as many new jobs overall as such companies added last year. Therefore, unless a record number of companies start up this year, Derderia will not break its record for new jobs created.
Which of the following is an assumption on which the argument relies?

(A) Each year, new companies starting up create more new jobs overall than do previously established companies.

The argument does not need this assumption.
(B) Companies established last year will not add a greater number of jobs overall this year than they did last year.

It has been referred in the argument.
(C) This year, the new companies starting up will not provide substantially more jobs per company than did new companies last year.

In this case, it is necessary for the logic chain of the argument, since if, this year, the new companies starting up add more new jobs per company than did new companies last year, the overall jobs created this year may still surpass those of last year. BA
(D) This year, the overall number of jobs created by previously established companies will be less than the overall number of jobs lost at those companies.

Jobs lost are outside the scope of the argument.
(E) The number of jobs created in the Derderian economy last year was substantially larger than the number of jobs lost last year.

The same to D.



20.
During the past year, Pro-Tect Insurance Company's total payout on car-theft claims has been larger than the company can afford to sustain. Pro-Tect cannot reduce the number of car-theft policies it carries, so it cannot protect itself against continued large payouts that way. Therefore, Pro-Tect has decided to offer a discount to holders of car-theft policies whose cars have antitheft devices. Many policyholders will respond to the discount by installing such devices, since the amount of the discount will within two years typically more than cover the cost of installation. Thus, because cars with antitheft devices are rarely stolen, Pro-Tect's plan is likely to reduce its annual payouts.

In the argument above, the two portions in boldface play which of the following roles?

(A) The first and the second are both evidence offered by the argument as support for its main conclusion.

The second is the main conclusion.
(B) The first presents a problem a response to which the argument assesses; the second is the judgment reached by that assessment.

The second is the main conclusion, and it is a judgment reached by the adoption of antitheft devices, while the first presents a problem for the reason of the adoption of antitheft devices. BA
(C) The first is the position the argument seeks to establish; the second is a judgment the argument uses to support that position.

The first is not the position the argument seeks to establish, neither does the second support the first.
(D) The first is a development that the argument seeks to explain; the second is a prediction the argument makes in support of the explanation it offers.

The second is not in support of the explanation, because the logic chain is reversed.
(E) The first presents a development whose likely outcome is at issue in the argument; the second is a judgment the argument uses in support of its conclusion about that outcome.

The second is not used to support the conclusion about the outcome.

作者: pennyz    时间: 2013-3-5 22:16
weaken or irrelevant
2:57s
a
conclusion:h is endanger because of ozone layer depletion
premise: h has no protection to uvb
rephrase: maybe other factor leads to h's extinction
作者: pennyz    时间: 2013-3-5 22:46
2:17s
weaken
d
premise: tall table can help customer see celebrity more clearly
        tall table leads to short meal time
conclusion: use tall table can raise profit
rephrase: maybe some other people will not visit tall table restraunt

1:57s
strengthen
a
premise: coast line built cost a lot
        farmers can not afford
conclusion:still think the trade is profitable
rephrase: maybe there is other service interested by farmers
2:17s
assumption
c
conclusion: unless new offer more jobs , can not set new job record
premise: old will not offer as many jobs as last year
rephrase: new company offer the same number of jobs as last year

3:18s
b
the annually pay out can't be resolved by the first
the second certificate that another method can resolve the problem
作者: Feelalive    时间: 2013-7-16 16:39
谢谢可爱卤煮小猪猪
作者: Elisha728    时间: 2013-9-5 07:26
9'28''
ADABD
作者: Elisha728    时间: 2013-9-7 13:34
4, With a record number of new companies starting up in Derderia and with previously established companies adding many jobs, a record number of new jobs were created last year in the Derderian economy.  This year, previously established companies will not be adding as many new jobs overall as such companies added last year.  Therefore, unless a record number of companies start up this year, Derderia will not break its record for new jobs created.
Which of the following is an assumption on which the argument relies?
(A) Each year, new companies starting up create more new jobs overall than do previously established companies.
-already stated in the passage “previously established companies will not be adding as many new jobs overall as such companies added last year.”

(B) Companies established last year will not add a greater number of jobs overall this year than they did last year
-already stated in the passage “previously established companies will not be adding as many new jobs overall as such companies added last year.”
(C) This year, the new companies starting up will not provide substantially more jobs per company than did new companies last year.

After negaton:

This year, the new companies starting up will provide substantially more jobs per company than did new companies last year.

If this is the case, Derderia will not break its record for new jobs created even without a record number of companies start up this year. Thus, the conclusion is overruled. so this is the correct assumption on which conclusion is based

(D) This year, the overall number of jobs created by previously established companies will be less than the overall number of jobs lost at those companies.
-already stated in the passage “previously established companies will not be adding as many new jobs overall as such companies added last year.”

(E) The number of jobs created in the Derderian economy last year was substantially larger than the number of jobs lost last year. Irrelevant

For assumption questions, use negation.

作者: Elisha728    时间: 2013-9-7 17:14
17, At present the Hollywood Restaurant has only standard-height tables.  However, many customers come to watch the celebrities who frequent the Hollywood, and they would prefer tall tables with stools because such seating would afford a better view of the celebrities.  Moreover, diners seated on stools typically do not stay as long as diners seated at standard-height tables.  Therefore, if the Hollywood replaced some of its seating with high tables and stools, its profits would increase.
The argument is vulnerable to criticism on the grounds that it gives reason to believe that it is likely that
(A) some celebrities come to the Hollywood to be seen, and so might choose to sit at the tall tables if they were available. Irrelevant
(B) the price of meals ordered by celebrities dining at the Hollywood compensates for the longer time, if any, they spend lingering over their meals. Irrlevant
(C) a customer of the Hollywood who would choose to sit at a tall table would be an exception to the generalization about lingering
First of all, understand the meaning of this option. An exception to the generalization about lingering= even customer who chooses to sit at a tall table may linger longer time. In this way, the profits will not increase.

(D) a restaurant's customers who spend less time at their meals typically order less expensive meals than those who remain at their meals longer.
Even if customers order less expensive means than those who remain at their meals longer, more rounds of customers may compensate for the price difference. So one can not tell whether the profit will increase or decrease.
(E) with enough tall tables to accommodate all the Hollywood's customers interested in such seating, there would be no view except of other tall tables. Irrelevant
作者: Elisha728    时间: 2013-9-7 17:15
bananazoo 发表于 2011-12-13 00:00
呀,发现最后一题,没有加粗……sorry~明天有电脑有电了就来加~囧………对了,大家17题是怎么做的?为什么 ...

17, At present the Hollywood Restaurant has only standard-height tables.  However, many customers come to watch the celebrities who frequent the Hollywood, and they would prefer tall tables with stools because such seating would afford a better view of the celebrities.  Moreover, diners seated on stools typically do not stay as long as diners seated at standard-height tables.  Therefore, if the Hollywood replaced some of its seating with high tables and stools, its profits would increase.
The argument is vulnerable to criticism on the grounds that it gives reason to believe that it is likely that
(A) some celebrities come to the Hollywood to be seen, and so might choose to sit at the tall tables if they were available. Irrelevant
(B) the price of meals ordered by celebrities dining at the Hollywood compensates for the longer time, if any, they spend lingering over their meals. Irrlevant
(C) a customer of the Hollywood who would choose to sit at a tall table would be an exception to the generalization about lingering
First of all, understand the meaning of this option. An exception to the generalization about lingering= even customer who chooses to sit at a tall table may linger longer time. In this way, the profits will not increase.

(D) a restaurant's customers who spend less time at their meals typically order less expensive meals than those who remain at their meals longer.
Even if customers order less expensive means than those who remain at their meals longer, more rounds of customers may compensate for the price difference. So one can not tell whether the profit will increase or decrease.
(E) with enough tall tables to accommodate all the Hollywood's customers interested in such seating, there would be no view except of other tall tables. Irrelevant
作者: sakurasong    时间: 2013-9-23 19:12
DEACB

4.
B: A populations declines aroung the world
P1zone layer depleted throughtout the last 50 ans
P2:Ozone blocks UV-B, which can damage genes
P3:Because of A's features,they are vulnerable to UV-B.
P4:Eggs lack the protection.
C:The primary cause of declining A is the deplation of ozone layer

A-Irrelevant


17.
B: The H has SH tables.
P1:C come to watch the celebrties who frequent the H and they prefer tall tables which offers a better view.
P2iners seated on stools do not stay as long as SH.
C: H gains more if it replaced some of its SH.


C-strengthen
作者: lyrsilvia    时间: 2013-9-23 19:18
E CACB
1. 31'
P:  amphibians lack hair, hide, or feathers to shield them, they are particularly vulnerable to UV-B radiation. their gelatinous eggs lack the protection of leathery or hard shells.
C:  primary cause of the declining amphibian population is the depletion of the ozone layer.
STRENGTHEN EXCEPT: the sun light is not the core cause
答案:E
(A) Of the various types of radiation blocked by atmospheric ozone, UV-B is the only type that can damage genes.----------it clearly suggest the UV-B is the cause of genes damage. strengthenCORRECT
(B) Amphibian populations are declining far more rapidly than are the populations of nonamphibian species whose tissues and eggs have more natural protection from UV-B.-----------also suggest the UV-B is to be blame
(C) Atmospheric ozone has been significantly depleted above all the areas of the world in which amphibian populations are declining.------------strengthen the atmospheric depleted is the cause
(D) The natural habitat of amphibians has not become smaller over the past century.-------------strengthen. this statement clearly shows the habitat is not changed, thus the decline is caused by UV-B
(E) Amphibian populations have declined continuously for the last 50 years.--------CORRECT. this statement just state the evidence. even though the ozone layer has been continuously depleted throughout the last 50 years, the simultaneous relation between the continuous decline and the layer depletion does not guarantee the actual relationship.
===============================================================
A fails to shed any light on the connection between the ozone depletion and the amphibian population decline. However, Answer E strengthens the argument by showing the decline of the amphibians has mirrored the decline of the ozone layer. 对比AE,E虽然不是百分百支持,像自己分析的一样有漏洞,但是E确实在一定程度上支持了C,而A在第一直觉觉得言他物的情况下,没有注意整体选项的暗含意思对比与整道题无关,误选了啊...
===============================================================
2. 33'
P: many customers come to watch the celebrities; they would prefer tall tables with stools; diners seated on stools typically do not stay as long as diners seated at standard-height tables
C: replaced some of its seating with high tables and stools, its profits would increase.
WEAKEN: tall tables≠more profits
答案:C
(A) some celebrities come to the Hollywood to be seen, and so might choose to sit at the tall tables if they were available-----------what celebrities choose to do does not matter the profits
(B) the price of meals ordered by celebrities dining at the Hollywood compensates for the longer time, if any, they spend lingering over their meals----------support the more viewer plan
(C) a customer of the Hollywood who would choose to sit at a tall table would be an exception to the generalization about lingering---------------CORRECT. an exception to the generalization about lingering means they would stay longer time, thus stools will not generate reduced stay in restaurant. this means only costs for replacement, but no advantages for profits. profits will not increase.
(D) a restaurant's customers who spend less time at their meals typically order less expensive meals than those who remain at their meals longer------------this statement just not illustrates the plan of replace seating with high tables and stools. even though it slightly prove that high tables and stools may reduce expensive meals orders. But it is possible that time reduced will be compensated by more customers.
(E) with enough tall tables to accommodate all the Hollywood's customers interested in such seating, there would be no view except of other tall tables--------------this just show the infeasibility of the plan. it does not clearly state the core of the problems

3. 25'
P: the only significant economic activity in these areas is small-scale coffee farming, and none of the coffee farmers make enough money to afford the monthly service fees that Calex would have to charge to make a profit
C:  making the service available to these farmers will be profitable.
SUPPORT: other factors that influence the profits may be taken into consideration
答案:A
(A) Currently, Caladian coffee farmers are forced to sell their coffee to local buyers at whatever price those buyers choose to pay because the farmers are unable to remain in contact with outside buyers who generally offer higher prices.----------CORRECT. means the service will enable the price to increase, which will help farmers to pay for the service
(B) In the coastal areas of Caladia where Calex proposes to introduce cellular telephone service, there is currently no fixed-line telephone service because fixed-line companies do not believe that they could recoup their investment.---------------means more cost and less attraction
(C) A cellular telephone company can break even with a considerably smaller number of subscribers than a fixed-line company can, even in areas such as the Caladian coast, where there is no difficult terrain to drive up the costs of installing fixed lines.-------------the comparison between cellular company and the fixed-line company is irrelevant
(D) Calex bases its monthly fees for cellular telephone service in a given region partly on the cost of installing the necessary equipment to provide the service there.---------------how C bases its monthly fees is irrelevant
(E) Calex has for years made a profit on cellular telephone service in Caladia's capital city, which is not far from the coastal region.------------how C makes profits form other city does not affect the profits from Caladia

4. 21'
P:  previously established companies will not be adding as many new jobs overall as such companies added last year
C: unless a record number of companies start up this year, Derderia will not break its record for new jobs created.
ASSUMPTION: the newly started company will not create more job than last year. the number of company≠the number of job
答案:C
(A) Each year, new companies starting up create more new jobs overall than do previously established companies.---------------if this true, even the number of new companies is less than last year, the jobs they created will be more than last year. Seriously undermine the conclusion
(B) Companies established last year will not add a greater number of jobs overall this year than they did last year.--------------this is just the premise stated in the question
(C) This year, the new companies starting up will not provide substantially more jobs per company than did new companies last year.-------------CORRECT. if new companies are providing more jobs per company than did last year, there will be more job created this year.
(D) This year, the overall number of jobs created by previously established companies will be less than the overall number of jobs lost at those companies.-------------we did not consider the jobs lost
(E) The number of jobs created in the Derderian economy last year was substantially larger than the number of jobs lost last year.--------------we did not consider the jobs lost

5. 43'
P: Pro-Tect cannot reduce the number of car-theft policies it carries, so it cannot protect itself against continued large payouts that way. Pro-Tect has decided to offer a discount to holders of car-theft policies whose cars have antitheft devices.  Many policyholders will respond to the discount by installing such devices, since the amount of the discount will within two years typically more than cover the cost of installation.
C: because cars with antitheft devices are rarely stolen, Pro-Tect's plan is likely to reduce its annual payouts.
BOLDFACE: first: evidence        second: conclusion
答案:B
(A) The first and the second are both evidence offered by the argument as support for its main conclusion.--------the second is not evidence
(B) The first presents a problem a response to which the argument assesses; the second is the judgment reached by that assessment.------------CORRECT
(C) The first is the position the argument seeks to establish; the second is a judgment the argument uses to support that position.---------------the first is not a position the argument seeks to establish
(D) The first is a development that the argument seeks to explain; the second is a prediction the argument makes in support of the explanation it offers.------------the first is not to explain
(E) The first presents a development whose likely outcome is at issue in the argument; the second is a judgment the argument uses in support of its conclusion about that outcome.------------the second is just the conclusion.


作者: jessie92730    时间: 2013-9-23 21:35
【offer大冒险】 2-3
BG: amphibian population, depletion of ozone layer not coincidentally
P: ozone- UV-B, amphibians vulnerable to UV-B
C: depletion of the ozone layer—declining amphibian population
推测答案:他因
答案:A 无关项

逻辑链:CACE(B)?

作者: Junjuo    时间: 2013-9-23 22:27
逻辑大冒险:Jun
DCABB
作者: w.melhere    时间: 2013-11-4 10:19
可不可以告诉我第二期的题比第一期的难了点...这3天做的都不太好呀... ><
1.2'30 错了。A和D犹豫了很久,还是没找到D为什么不对... 看了解释...
D,其实是加强项,排除他因。Amphibian的减少不是因为栖息地变小(比如因为其他人为原因等)。

2.2'00 嗯 也错。
C:an exception to the generalization about lingering means they would stay longer time, thus stools will not generate reduced stay in restaurant.
而D有一个gap,可能停留时间长的人会order更贵的东西,但是如果停留时间短的人比停留长的人更多,那么即使order比较便宜也会比较多profit。 不能单纯从数量上判断,没有价格无法比较。

3.1'54
4.1'14
5.1'58
作者: yuehuasunday    时间: 2013-11-23 16:45
2-3

1.

Background: Amphibian populations are declining in numbers
worldwide.

Premise: Not coincidentally, the earth’s ozone
layer has been continuously depleted throughout the
last 50 years. Atmospheric ozone blocks UV-B, a
type of ultraviolet radiation that is continuously
produced by the sun, and which can damage genes.
Because amphibians lack hair, hide, or feathers to
shield them, they are particularly vulnerable to UV-B
radiation. In addition, their gelatinous eggs lack the
protection of leathery or hard shells.

Conclusion: Thus, the
primary cause of the declining amphibian population
is the depletion of the ozone layer.

(D) Correct

2.

Background: At present the Hollywood Restaurant has only standard-height tables.

Premise: However, many customers come to watch the celebrities who frequent the Hollywood, and they would prefer tall tables with stools because such seating would afford a better view of the celebrities.  Moreover, diners seated on stools typically do not stay as long as diners seated at standard-height tables.

Conclusion: Therefore, if the Hollywood replaced some of its seating with high tables and stools, its profits would increase.

(D) Correct

3.

Background: The Calex Telecommunications Company is planning to introduce cellular telephone service into isolated coastal areas of Caladia, a move which will require considerable investment.

Premise: However, the only significant economic activity in these areas is small-scale coffee farming, and none of the coffee farmers make enough money to afford the monthly service fees that Calex would have to charge to make a profit.

Conclusion: Nevertheless, Calex contends that making the service available to these farmers will be profitable.

(A) Correct

4.

Background: With a record number of new companies starting up in Derderia and with previously established companies adding many jobs, a record number of new jobs were created last year in the Derderian economy.

Premise: This year, previously established companies will not be adding as many new jobs overall as such companies added last year.

Conclusion: Therefore, unless a record number of companies start up this year, Derderia will not break its record for new jobs created.

© Correct

5.

Background: During the past year, Pro-Tect Insurance Company's total payout on car-theft claims has been larger than the company can afford to sustain.

Premise:  Pro-Tect cannot reduce the number of car-theft policies it carries, so it cannot protect itself against continued large payouts that way.

Conclusion: Thus, because cars with antitheft devices are rarely stolen, Pro-Tect's plan is likely to reduce its annual payouts.

(B) Correct
作者: 苏错    时间: 2013-11-24 21:35
11.24
[精练]
background:the ozone layer blocks harmful UV-B
radiation
conclusion:primary cause of the declining amphibian population
is the depletion of the ozone layer
原因类的分析 求非支持类
A. 可能未考虑到的
B.strengthen
C.strengthen
D.strengthen
E.strengthen

B.C.D.E证明了相关性

[逻辑链]
1.weaken  C
2.support  A
3.assumption  C
4.boldface B

作者: irenetopia    时间: 2013-12-7 00:02
【精练】4. eprimary cause of the Amphibian populations are declining in numbersworldwide. Not coincidentally, the earth’s ozonelayer has been continuously depleted throughout thelast 50 years. Atmospheric ozone blocks UV-B, atype of ultraviolet radiation that is continuouslyproduced by the sun, and which can damage genes.Because amphibians lack hair, hide, or feathers toshield them, they are particularly vulnerable to UV-Bradiation. In addition, their gelatinous eggs lack theprotection of leathery or hard shells. Thus, declining amphibian populationis the depletion of the ozone layer.Each of the following, if true, would strengthen theargument EXCEPT:
【做错了】A 的逐步灭绝和臭氧层耗尽同时发生
臭氧层含有UVB,会伤害基因,A又缺乏保护
所以!A 的灭绝是因为——臭氧层耗尽了!
问weaken或irrelevant
分析,从上面来说,臭氧层减少对A来说是好事啊
(A) Of the various types of radiation blocked byatmospheric ozone, UV-B is the only typethat can damage genes.——如果只有UVB伤害A那么臭氧层变少=UVB减少,不会让A绝种,weaken(B) Amphibian populations are declining far morerapidly than are the populations ofnonamphibian species whose tissues andeggs have more natural protection fromUV-B.(C) Atmospheric ozone has been significantlydepleted above all the areas of the world inwhich amphibian populations are declining.(D) The natural habitat of amphibians has notbecome smaller over the past century.(E) Amphibian populations have declinedcontinuously for the last 50 years.——为什么我会选这个,望天……

【逻辑链】17.At present the Hollywood Restaurant has only standard-height tables.  However, many customers come to watch the celebrities who frequent the Hollywood, and they would prefer tall tables with stools because such seating would afford a better view of the celebrities.  Moreover, diners seated on stools typically do not stay as long as diners seated at standard-height tables.  Therefore, if the Hollywood replaced some of its seating with high tables and stools, its profits would increase.The argument is vulnerable to criticism on the grounds that it gives reason to believe that it is likely that
逻辑链:高椅子,坐的时间少,这样流动性大,就可以增加利润
问weaken:两个:一个是客人因为喜欢看明星赖着不走
二是因为主要目的是看明星,所以不会点很贵的菜甚至是点小吃
(A) some celebrities come to the Hollywood to be seen, and so might choose to sit at the tall tables if they were available——irrelevant(B) the price of meals ordered by celebrities dining at the Hollywood compensates for the longer time, if any, they spend lingering over their meals——irrelevant(C) a customer of the Hollywood who would choose to sit at a tall table would be an exception to the generalization about lingering——留着,客人赖着不走,但是会不会点的也多呢(D) a restaurant's customers who spend less time at their meals typically order less expensive meals than those who remain at their meals longer——bingo!两个都有!强于C(E) with enough tall tables to accommodate all the Hollywood's customers interested in such seating, there would be no view except of other tall tables——irrelevant18.The Calex Telecommunications Company is planning to introduce cellular telephone service into isolated coastal areas of Caladia, a move which will require considerable investment.  However, the only significant economic activity in these areas is small-scale coffee farming, and none of the coffee farmers make enough money to afford the monthly service fees that Calex would have to charge to make a profit.  Nevertheless, Calex contends that making the service available to these farmers will be profitable.Which of the following, if true, provides the strongest support for Calex's contention?
逻辑链:CT想要引进电话服务
但是唯一的经济支持——咖啡农,没有钱支付这个费用,也就是不会有profit
但是CT非要认为给农民提供这个服务会盈利
问解释:改变了因的因素,也就是为什么会盈利,农民什么情况下可以接受这个电话并且支付得起?
(A) Currently, Caladian coffee farmers are forced to sell their coffee to local buyers at whatever price those buyers choose to pay because the farmers are unable to remain in contact with outside buyers who generally offer higher prices.——bingo!因为有的电话,就能和outside buyer联系,农民就有钱,就能支付费用,良性循环(B) In the coastal areas of Caladia where Calex proposes to introduce cellular telephone service, there is currently no fixed-line telephone service because fixed-line companies do not believe that they could recoup their investment.——irrelevant(C) A cellular telephone company can break even with a considerably smaller number of subscribers than a fixed-line company can, even in areas such as the Caladian coast, where there is no difficult terrain to drive up the costs of installing fixed lines.——weaken(D) Calex bases its monthly fees for cellular telephone service in a given region partly on the cost of installing the necessary equipment to provide the service there.——weaken(E) Calex has for years made a profit on cellular telephone service in Caladia's capital city, which is not far from the coastal region.——irrelevant19.With a record number of new companies starting up in Derderia and with previously established companies adding many jobs, a record number of new jobs were created last year in the Derderian economy.  This year, previously established companies will not be adding as many new jobs overall as such companies added last year.  Therefore, unless a record number of companies start up this year, Derderia will not break its record for new jobs created.Which of the following is an assumption on which the argument relies?
逻辑链:去年,D有很多新公司开业,有大量职位,因此就业率有了record
但是今年,去年的公司没有增加比去年多的新的职位了
所以除非有新的公司开业,不然今年就不能打破去年的就业record
问假设:是今年新开的公司增加的职位不比去年的少?否则开了也不能创新高(A) Each year, new companies starting up create more new jobs overall than do previously established companies.——bingo!(B) Companies established last year will not add a greater number of jobs overall this year than they did last year.——重复前提(C) This year, the new companies starting up will not provide substantially more jobs per company than did new companies last year.——weaken(D) This year, the overall number of jobs created by previously established companies will be less than the overall number of jobs lost at those companies.——irrelevant(E) The number of jobs created in the Derderian economy last year was substantially larger than the number of jobs lost last year.——irrelevant啊啊啊啊!
又错了!!!!为什么!!!!
其他人的解释
又看到这道题了,再看到自己的回复时小小惊了一下,呵呵,现在这题想明白了,贴出来:再做还是错得一样,现在理解下来:工作数量有两个因素决定1:老企业提供的工作岗位;2.新建成企业提供的工作岗位。原题是说第一个因素今年和去年没有变化,那就看第二个因素。结论中注意“a record number”字样,意思是原题的结论要是新企业数量必须有个创纪录的数量上升,言下之意就是如果非得要企业数量上升才能使总数上升的话,那么就该有个假设:单个企业提供的工作数量下降。就比如a*b=c,已知c是上升的,如果要a必须上升,那么只有b下降。A呢,我觉得是上面等式中c上升,和a、b本身再怎么样也就没什么关系了,即不能推到原题的结论

20.During the past year, Pro-Tect Insurance Company's total payout on car-theft claims has been larger than the company can afford to sustain.  Pro-Tect cannot reduce the number of car-theft policies it carries, so it cannot protect itself against continued large payouts that way.  Therefore, Pro-Tect has decided to offer a discount to holders of car-theft policies whose cars have antitheft devices.  Many policyholders will respond to the discount by installing such devices, since the amount of the discount will within two years typically more than cover the cost of installation.  Thus, because cars with antitheft devices are rarely stolen, Pro-Tect's plan is likely to reduce its annual payouts.In the argument above, the two portions in boldface play which of the following roles?
第一部分是考虑中的一个因素,第二个是最后的评价?(A) The first and the second are both evidence offered by the argument as support for its main conclusion.(B) The first presents a problem a response to which the argument assesses; the second is the judgment reached by that assessment.(C) The first is the position the argument seeks to establish; the second is a judgment the argument uses to support that position.(D) The first is a development that the argument seeks to explain; the second is a prediction the argument makes in support of the explanation it offers.(E) The first presents a development whose likely outcome is at issue in the argument; the second is a judgment the argument uses in support of its conclusion about that outcome.(A) The first and the second are both evidence offered by the argument as support for its main conclusion.(B) The first presents a problem a response to which the argument assesses; the second is the judgment reached by that assessment.(C) The first is the position the argument seeks to establish; the second is a judgment the argument uses to support that position.(D) The first is a development that the argument seeks to explain; the second is a prediction the argument makes in support of the explanation it offers.(E) The first presents a development whose likely outcome is at issue in the argument; the second is a judgment the argument uses in support of its conclusion about that outcome.
作者: cyndichiang    时间: 2014-4-23 10:55
精炼:
premise: A is vulnerable to UV-B radiation because of lack of leathry and hard shells
conclusion: the primary cause of the declining A  population is the depletion of the ozone layer.
                Conclusion--------------------->Evidence
         declining population------------> depletion of ozone layer
Q:stengthen EXCEPT
prephrase: irrelevant or weaken statement can be the answer
(A) Of the various types of radiation blocked by
atmospheric ozone, UV-B is the only type
that can damage genes.----Correct,irrelevant, whether what kind of radiation can affect the A, the depletion is the cause of those radiation,so whether the kinds of radiation is irrelevant 这里分析的与答案有出入,答案说的是已经默认为是UV-B了,所以感觉有点像是质疑前提的,但是质疑前提的与结论无关,因为前提是文章默认的事实
Because the argument is concerned with the damage done by UV-B radiation, the fact that UV-B is the only damaging type of radiation blocked by ozone is irrelevant.
(B) Amphibian populations are declining far more
rapidly than are the populations of
nonamphibian species whose tissues and
eggs have more natural protection from
UV-B.------------------strengthen; 有因有果的例证。depletion&vulnerable-->UV-B-->decline more
(C) Atmospheric ozone has been significantly
depleted above all the areas of the world in
which amphibian populations are declining.-------strengthen; this statement confirm the fact of depletion
(D) The natural habitat of amphibians has not
become smaller over the past century.-------strengthen;this statement eliminates a factor that can also lead to the deline population---->否定了一个削弱的因素
(E) Amphibian populations have declined
continuously for the last 50 years.-------strengthen; this statement confirms the conclusion
弱弱的问一下。。那个答案中的TYPE(CAE)啥的是神马意思。。。


         
作者: cyndichiang    时间: 2014-4-23 11:19
逻辑连
1. 39‘’
premise:customers would prefer tall tables with stools
premise:the time dinners stay doesn't change
conclusion: the restuarant will increase profits by replacing the standard tables with tall one
Q:weaken
prephrase:  the cost of tall tables is much more than standard table and replacement fee is high


2.37''
premise:coffee farmers cannot afford the telecommunication service fees because of small-scale plant
conclusion: making the service available to these farmers will be profitable
Q:support
prephrase: the use of the cellers can help farmers to do business with outsiders thus gain profits

3.26''
premise: previous compainies will not provide more new jobs than it did in the last year
conclusion:unless a record number of companies start up this year, no more new jobs will be created than did previous year
Q:Assumption
prephrase: if the some older companies established years ago can create more jobs?错了错了--->因为文章中已经说了去年是一个record所以之前的不可能更多
                conclusion说的是新公司的数量,但是没有说新公司是否增加的新工作比去年的多啊--->
                                             the number companies* jobs/per company=whole jobs of new companies

4. 1'04''
Pro-Tect Insurance Company's total payout on car-theft claims has been larger than the company can afford to sustain. ---->evidence
Pro-Tect cannot reduce the number of car-theft policies it carries, so it cannot protect itself against continued large payouts that way.  ---->evidence
Therefore, Pro-Tect has decided to offer a discount to holders of car-theft policies whose cars have antitheft devices.  ---->subconclusion( a proposal)
Many policyholders will respond to the discount by installing such devices, since the amount of the discount will within two years typically more than cover the cost of installation. ---->evidence
Thus, because cars with antitheft devices are rarely stolen, Pro-Tect's plan is likely to reduce its annual payouts.------>conclusion



作者: May97    时间: 2018-3-13 08:04
T1: 48S
P: Amphibian populations are declining in numbers worldwide.
P: the earth’s ozone layer has been continuously depleted throughout the last 50 years.
P: Atmospheric ozone blocks UV-B, a type of ultraviolet radiation that is continuously produced by the sun, and which can damage genes.
P: Because amphibians lack hair, hide, or feathers to shield them, they are particularly vulnerable to UV-B radiation.
P: In addition, their gelatinous eggs lack the protection of leathery or hard shells.
C: Thus, the primary cause of the declining amphibian population is the depletion of the ozone layer.
STRENGTH X:
(A) Of the various types of radiation blocked by atmospheric ozone, UV-B is the only type that can damage genes.--->R
(B) Amphibian populations are declining far more rapidly than are the populations of nonamphibian species whose tissues and eggs have more natural protection from UV-B.
(C) Atmospheric ozone has been significantly depleted above all the areas of the world in which amphibian populations are declining.
(D) The natural habitat of amphibians has not become smaller over the past century.--->IRREVERENT
(E) Amphibian populations have declined continuously for the last 50 years.

Answer choice (A): This is the correct answer. The answer fails to shed any light—positive or negative—on the connection between the ozone depletion and the amphibian population decline. Because the argument is concerned with the damage done by UV-B radiation, the fact that UV-B is the only damaging type of radiation blocked by ozone is irrelevant.
Answer choice (D): This was the answer most frequently chosen by test takers. This answer choice strengthens the argument by eliminating an alternate cause for the effect (Type A). Had the natural habitat become smaller over the years (from say, human encroachment or climatic change) then that shrinkage would have offered an alternate explanation for the decline in the amphibian population. By eliminating the possibility of habitat shrinkage, the stated cause in the argument is strengthened.

T: 43S
BG: At present the Hollywood Restaurant has only standard-height tables.  
P: However, many customers come to watch the celebrities who frequent the Hollywood, and they would prefer tall tables with stools because such seating would afford a better view of the celebrities.  
P: Moreover, diners seated on stools typically do not stay as long as diners seated at standard-height tables.  
C: Therefore, if the Hollywood replaced some of its seating with high tables and stools, its profits would increase.
WEAKEN: THE NUMBER OF COUSTOMS WILL INCREASE, BUT THE REVENUE WILL DECREASE
(A) some celebrities come to the Hollywood to be seen, and so might choose to sit at the tall tables if they were available--->IRREVERENT
(B) the price of meals ordered by celebrities dining at the Hollywood compensates for the longer time, if any, they spend lingering over their meals--->SUPPORT
(C) a customer of the Hollywood who would choose to sit at a tall table would be an exception to the generalization about lingering--->SUPPORTR
(D) a restaurant's customers who spend less time at their meals typically order less expensive meals than those who remain at their meals longer--->R???
(E) with enough tall tables to accommodate all the Hollywood's customers interested in such seating, there would be no view except of other tall tables--->IRREVERENT

T: 36S
P: The Calex Telecommunications Company is planning to introduce cellular telephone service into isolated coastal areas of Caladia,
P: a move which will require considerable investment.  
P: However, the only significant economic activity in these areas is small-scale coffee farming, and none of the coffee farmers make enough money to afford the monthly service fees that Calex would have to charge to make a profit.  
C: Nevertheless, Calex contends that making the service available to these farmers will be profitable.
SUPPORT:
(A) Currently, Caladian coffee farmers are forced to sell their coffee to local buyers at whatever price those buyers choose to pay because the farmers are unable to remain in contact with outside buyers who generally offer higher prices.--->R
(B) In the coastal areas of Caladia where Calex proposes to introduce cellular telephone service, there is currently no fixed-line telephone service because fixed-line companies do not believe that they could recoup their investment.--->IRREVERENT
(C) A cellular telephone company can break even with a considerably smaller number of subscribers than a fixed-line company can, even in areas such as the Caladian coast, where there is no difficult terrain to drive up the costs of installing fixed lines.--->IRREVERENT
(D) Calex bases its monthly fees for cellular telephone service in a given region partly on the cost of installing the necessary equipment to provide the service there.--->IRREVERENT
(E) Calex has for years made a profit on cellular telephone service in Caladia's capital city, which is not far from the coastal region.--->IRREVERENT

T: 28S
P: With a record number of new companies starting up in Derderia and with previously established companies adding many jobs, a record number of new jobs were created last year in the Derderian economy.  
P: This year, previously established companies will not be adding as many new jobs overall as such companies added last year.  
C: Therefore, unless a record number of companies start up this year, Derderia will not break its record for new jobs created.
FA: HOW ABOUT THE NEW COMPANIES?
(A) Each year, new companies starting up create more new jobs overall than do previously established companies.--->WEAKEN
(B) Companies established last year will not add a greater number of jobs overall this year than they did last year.--->SAME AS PREMISE
(C) This year, the new companies starting up will not provide substantially more jobs per company than did new companies last year.--->R
(D) This year, the overall number of jobs created by previously established companies will be less than the overall number of jobs lost at those companies.--->IRREVERENT
(E) The number of jobs created in the Derderian economy last year was substantially larger than the number of jobs lost last year.--->IRREVERENT

T: 53S
During the past year, Pro-Tect Insurance Company's total payout on car-theft claims has been larger than the company can afford to sustain.  Pro-Tect cannot reduce the number of car-theft policies it carries, so it cannot protect itself against continued large payouts that way.  Therefore, Pro-Tect has decided to offer a discount to holders of car-theft policies whose cars have antitheft devices.  Many policyholders will respond to the discount by installing such devices, since the amount of the discount will within two years typically more than cover the cost of installation.  Thus, because cars with antitheft devices are rarely stolen, Pro-Tect's plan is likely to reduce its annual payouts.
BF:
(A) The first and the second are both evidence offered by the argument as support for its main conclusion.
(B) The first presents a problem a response to which the argument assesses; the second is the judgment reached by that assessment.--->R
(C) The first is the position the argument seeks to establish; the second is a judgment the argument uses to support that position.
(D) The first is a development that the argument seeks to explain; the second is a prediction the argument makes in support of the explanation it offers.
(E) The first presents a development whose likely outcome is at issue in the argument; the second is a judgment the argument uses in support of its conclusion about that outcome.--->???



作者: echo-LUO    时间: 2018-10-14 10:27
马上就要上考场了,最后来练练手。希望CD能够保佑我,不求分手,只求比一战强一些,尤其是verbal(这段时间事情杂七杂八的来,其实没有好好复习,惭愧)。
精练:
逻辑链:a物种数量减少——>臭氧层破坏——>UVB射线进入——>a没有防护措施阻挡,射线UVB破坏egg——>conclusion:臭氧破坏是a减少的原因
(A) Of the various types of radiation blocked by atmospheric ozone, UV-B is the only type that can damage genes.?疑问,似乎无关
(B) Amphibian populations are declining far more rapidly than are the populations of nonamphibian species whose tissues and eggs have more natural protection from UV-B. 加强
(C) Atmospheric ozone has been significantly depleted above all the areas of the world in which amphibian populations are declining. 加强
(D) The natural habitat of amphibians has not become smaller over the past century. ?似乎可以,因为削弱了外因——>不是因为栖息地减少而造成a减少?有些牵强
(E) Amphibian populations have declined continuously for the last 50 years. 加强
最后选择A,感觉D或多或少还有点关系

做对了!!!
希望考场CR也能做对!!!
祈祷

作者: 云栈    时间: 2019-11-14 15:05
1.        40s
P: 世界各地的两栖动物数量正在减少。并非巧合的是,过去50年来,地球的臭氧层一直在不断消耗。
P: 大气臭氧会阻止UV-B,这是一种由太阳连续产生的紫外线,会破坏基因。
P: 由于两栖动物缺乏毛发,皮革或羽毛来遮挡它们,因此它们特别容易受到UV-B辐射的伤害。
P: 另外,它们的胶状鸡蛋缺乏对皮革或硬壳的保护。
C: 因此,两栖动物数量下降的主要原因是臭氧层的消耗。
推测:无关选项;amphibian在ozone layer特别稀少的地方的数量和其他地方差不多
选项分析:A
A: 题上说的是A的数量和ozone有关,并没有说跟UB-V有关
B:帮助解释A的数量降低和UV-B有关
C: 帮助解释A的数量并不是因为某个地区的气候或者其他特殊性而降低
D: 帮助解释A的数量不是因为自然栖息地变少才降低的
E: A的数量开始降低的时间跟ozone消失的时间一样

2.        38s
P: 目前,好莱坞餐厅只有标准高度的桌子。
P: 但是,许多顾客来观看经常光顾好莱坞的名人,他们更喜欢带凳子的高脚桌,因为这样的座位可以更好地欣赏名人。
P: 此外,坐在凳子上的食客通常不会停留在坐在标准高度桌子上的食客停留的时间。
C: 因此,如果好莱坞用高脚桌和凳子代替一些座位,其利润将会增加。
推测:吃饭的人可能因为凳子太高了就不吃了;换了高凳子在别的方面会降低营收
选项分析:C
【这题选错了,问题的意思翻译过来就是,这个推论是错的,因为他有一个假设不正确,问这个假设是什么。 C: a customer of the Hollywood who would choose to sit at a tall table would be an exception to the generalization about lingering 的意思是在说一般人不会坐在高凳子上面,也就意味着选择做高凳子的都是为了看名人的,那么他们就不可能很快就走。D虽然说吃的时间短的点的便宜,但是如果翻桌率高可能收益更高
这个问题的要点是根据文中提供的信息而不是举出其他原因】

3.        34s
P: Calex电信公司计划将蜂窝电话服务引入Caladia的偏远沿海地区,此举将需要大量投资。
P: 但是,这些地区唯一的重要经济活动是小规模咖啡种植,没有任何咖啡种植者能赚到足够的钱来支付Calex为赚钱而必须收取的每月服务费。
C: 然而,Calex认为向这些农民提供服务将是有利可图的。
推测:可以通过电话订单出口咖啡,公司charge打入电话的人的钱
选项分析:A
【这道题问的是安装电话可以让农民获利,读题读成了可以让电话公司获利,所以选错了,还是要认真看题】

4.        32s
P: 随着在德里达(Derderia)成立的新公司数量创纪录,并且以前成立的公司增加了很多工作,去年在德德尔经济中创造了创纪录数量的新工作。
P: 今年,以前成立的公司将不会像去年增加的那样增加那么多新工作。
C: 因此,除非今年有创纪录的公司成立,否则Derderia不会打破创造新工作岗位的记录。
推测:新开的公司会从当地开放很多岗位在当地,比之前的公司还要多
选项分析:C
【原题说除非今年新的公司比去年多,否则不会超记录,这中间的逻辑漏洞就是如果今年新的公司比去年少,但是开放的岗位比去年多很多,也有可能超记录,所以要补充说明今年每个新公司开的岗位不必去年开的多,所以选C】

5.        58s
P:在过去的一年中,Pro-Tect保险公司在汽车盗窃索赔方面的总支出超出了该公司承受的范围。
C:Pro-Tect无法减少其实施的盗车政策的数量,因此它无法保护自己免遭持续的巨额支出。
P:因此,Pro-Tect已决定为拥有防盗装置的汽车盗窃保单持有人提供折扣。
P: 许多保单持有人将通过安装这种设备来响应折扣,因为折扣金额通常会在两年内超过安装成本。
C: 因此,由于带有防盗装置的汽车很少被盗,因此Pro-Tect的计划很可能会减少其年度支出。
推测:第一部分是陈述了一个judgement,用来总结前面的事实。第二部分是对公司实行的政策的一个结论
选项分析:B
【main conclusion是最后一句,D不对因为第一部分不是a development that the argument seeks to explain; “position argument is trying to establish”=“main conclusion”】

作者: Lincy123    时间: 2020-6-4 09:03
P:amphibian没有毛发遮挡UV-B
C:他们数量减少的主要原因是臭氧层破坏
Not strengthen:他因
A) 只有UV_B会破坏基因。排除其他射线 correct
没有提到臭氧破坏和数量下降的关联,只说了UV-B是坏的。
B) 不受UV_B的non-amphibian数量减少得更慢。Strengthen
C) 在am减少的地方臭氧层都被破坏了。Strengthen
D)自然栖息地没有变得更少,strengthen
E) 过去50年am数量持续减少。Correct(x)

P:把正常桌子换成高脚桌
C:利润会上升
Weaken
A) 一些名人会选择做高脚桌。无关
B) 名人点的餐要吃更久时间。
C) 坐在高脚桌的客人会是一个例外 correct
转载:https://forum.chasedream.com/thread-565901-1-1.html
Premise自相矛盾!
Premises:
1) Customers come to Hollywood Restaurant to watch the celebrities so customrs would prefer tall tables to get a better view.
2) Diners seated on stools typically stay a shorter time than diners on regular seats.

Conclusion:
If the Hollywood replaced some of its seating with high tables and stools, its profits would increase.

Basically, the argument says that stools would attract more customers and customers sitting on stools turn over quickly.  Therefore, profits would be up.  Wait a minute.  Based on premise 1, if the customers are attracted to the restaraunt because they want to see celebrities, shouldn't they stay LONGER than normal customers? If so, it runs contrary to premise 2 which describes a general trend in customer's lingering behavior. The customer attracted might sit on the stools for a LONNNNNNNNNNNNNNG time without spending much on food. No turnover, no money!
D)花更少时间在餐点的客人会点更便宜的餐。Correct(x)
E) 如果所有人都坐高脚桌,就没有视野优势。无关

咖啡农买不起月费,但C说提供给农民服务会赚钱
Strengthen
A) 咖啡农没有议价能力,因为他们联系不到外界的高价顾客。Correct
B) 这个地方没有固定电话服务,因为收不回投资。
C) 公司可以收支平衡
D)月费的基础在安装设备上
E) 在隔壁首都C可以赚钱

P:如果今年没有创纪录数量的创业公司
C:新工作记录不会被打破
Assumption:一个公司不会招多个岗位
A)每年新创公司比现有公司创造更多岗位。无关
B)去年创立的公司在今年不会招更多人。
C)今年每个新创公司不会比去年平均招的人更多。Correct

B1:保险公司不能减少防盗保险,所以会一直赔钱
B2:因为有防盗设备的车很少被偷,保险公司很可能减少每年的开销
B1是背景,B2是p+c结论
A) B1 B2是论据
B) B1呈现一个问题;B2是能达到assessment的一个判断 correct
C) B1是想达到的
D)B1是想解释的
E) B1的结果带出问题,B2支持这个结果






欢迎光临 ChaseDream (https://forum.chasedream.com/) Powered by Discuz! X3.3